Calculus I Math 213
Calculus I Math 213
TABLE OF CONTENTS
Contents Page
Unit 1: Functions 1
Unit 2: Limits 17
1
Unit 1: functions
CONTENT
1.0 Aims and Objectives
1.1 Introduction
1.2 Definition
1.3 Types of Functions
1.4 Graphs of functions
1.5 Combinations of Functions
1.6 Summary
1.7 Answers to Check Your Progress Questions
1.8 Model Examination Questions
1.1 INTRODUCTION
In calculus we are most often interested in related variables. For example, the distance an object
moves is related to its speed, a person’s weight is related to his age, and so on. There is an
especially important kind of relation between variable which characterizes a function. In this unit
we review the notion of function.
2
1.2 DEFINITION
A function is a correspondence between two sets of numbers that has the property, to each number
in the first set, called the domain of the function, there corresponds one and only one number in
the second set.
Definition: Let x and y be nonempty sets. Let f be a set of ordered pairs (x, y) with x X and y
Y. then f is a function from X to Y if to every x X there is assigned a unique
y Y.
Note: The collection of all first elements x of the pairs (x, y) in is called the domain of , and
will be denoted by Df. The range of f, denoted by R, is the set of all second elements y of the
pairs (x, y) in . The definition states that every x in X is the first entry of exactly one pairs (x, y)
in .
Example 1.2.1 Let X be the set of all triangles and let Y be the set of all real numbers. Let be
the set of ordered pairs (x, y) where
X is any triangle, and
Y is the area enclosed by that triangle, measured in appropriate units.
To each triangle X, there corresponds a unique real number Y that represents its area; so is a
function. The domain of is all of X; the range consists of all positive real numbers.
FUNCTION NOTATION
To be explicit, let us denote domain values (independent variable) by X, range values (dependent
variable) by Y and the function by . the value of X and Y are numbers, but is something more
abstract. can be thought as a rule or procedure establishing a correspondence between the value
of X and those of Y and assigning a unique value of Y to any given value of X. we express this
symbolically by writing
Y = f (x)
3
Which is read as “y equals of x”. If x is assigned any particular value c. the corresponding value
of y is denoted by f(c) and is called the value of at c.
Example 1.2.2 Let f be the function which takes the square of a given number x. then
f (x) = x2
Some values of the function f (x) are
f (1) = 12 = 1, f (-2) = (-2)2 = 4, f (10) = 102 = 100
Note Whenever a function is given by an explicit formula with no information about the values of
the independent variable, it is understood that the domain of the function is the largest set of values
for which the formula makes sense.
If f (x) = 1 x2
D is the interval –1 x 1, since 1 – x2 is negative outside this interval.
1
Example 1.2.3 Let f (x) =
x 9
2
SOLUTION
To find f (c) we substitute x = c into the formula i.e
1 1 1 1
f (0) = f (2) =
(0) 9
2
9 2 9
2
5
1 1 1 1
f (1) = f (-3) =
1 9
2
8 (3) 9 0
2
D is x2 – 9 0 x 3
i.e the domain of f (x) consists of all real numbers except 3.
4
a) h (0) b) h (1) c) h (100) d) h (-99) e) h ()
3. Find the domain of the given function
x ( x 1) 1
a) f (x) = b) f (x) = c) f (x) = 3
x 1
x x x
x 1
d) f (x) = 16 x 2 e) f (x) =
2 x
1 1 -2 0
2 4 0
3 9 2 4
1 1 1
2 2 2
3 3 3
1-1 function not one to one function
Definition 1.3.1
A function f is one to one if and only if a b implies f (a) f (b), whenever a, b D
Solution
a) To show that a function is not one to one, find two different elements in the domain of the
function that produce the same range element.
5
b) for any a, b D
a b f (a) f (b)
2a – 1 2b – 1
2a 2b
ab
Note: A function is one to one if and only if each horizontal line intersects the graph of the
function in at most one point.
6
7
II. On-to function
Definition 1.3.2
A function f: A B is an onto function if and only if every element of B is an image of some
element in A. i.e the range of is B
Example 1.3.3
f1 f2 f3
a) A B b) A B c) A B
1 a 1 a 1 a
2 b 2 b 2 b
3 c 3 c 3 c
4 d 4 d 4
Note: one to one function may or may not be onto function and vice versa.
Definition 1.3.3 Suppose f: A B is both one to one and onto function, then it is called one to
one correspondence.
Example 1.3.4
f
1 a
2 b
3 c
Solution
(1) Check f is one to one
Let x1, x2 D and x1 x2
W.T.S x1 x2 f (x1) f (x2)
2x1 2x2
x1 x2 f is 1 – 1 function.
y y
f( )=s( )=y
2 2
f is onto function
Definition 1.3.4
1. A function f is increasing on its domain if f (a) > f (b) whenever a > b, a, b D.
2. A function f is decreasing on the D if f (a) < f (b) whenever a > b; a, b D
3. A function f is a constant function if f (a) = f (b) for all a, b D.
9
1.4 GRAPHS OF FUNCTION
The graph of a function f is the graph of the equation y = f (x). It consists of the points in the
Cartesian plane whose coordinates (x, y) are input-output pairs of f.
Note every curve you draw is the graph of a function. A function f can have only one value f (x)
for each x in its domain, so no vertical line can intersect the graph of a function more than once.
If a is in the domain of a function f, then the vertical line x = a will intersect the graph of f in the
single point (a, f (a)).
Solution
Step 1 Make a table of xy-pairs that satisfy the function rule, in this case the equation
y = x2
X –2 –1 0 1 2
Y = x2 4 1 0 1 4
Step 2 Plote the points (x, y) whose coordinate appear in the table on the xy-plane
(2, 4) * 4 * (2, 4)
(-1, 1) * 1 * (1, 1)
-2 -1 1 2 X
Step 3 Draw a smooth curve through the plotted points. Label the curve with its equation.
10
How do we know the graph of y = x2 doesn’t look like one of these curves?
To find out we could plot more points. But how would we then connect them? The basic question
still remains: How do we know for sure what the graph look like between the points we plot? The
answer lies in calculus, as we will see in the coming units there will be a mathematical tool called
the derivative to find the curve’s shape between plotted points. Mean while we will have to settle
for plotting points and connecting them as best we can.
The graph of an even function y = f (x) is symmetric about the Y-axis. Since f (-x) =
f (x), the point (x, y) lie on the graph if and only if the point (-x, y) lie on the graph. Once we know
the graph on one side of the Y-axis, we automatically know it on the other side.
11
Symmetry about the Y-axis if (x, y) is on the graph, so is (-x, y)
A function y = f (x) is odd if f (-x) = -f (x) for every number x in the domain of f. again, both x
and –x must lie in the domain of f. the function f (x) = x3 is odd because f (-x) =
(-x)3 = -x3 = -f (x).
The graph of an odd function y = f (x) is symmetric about the origin. Since f (-x) = -f (x), the point
(x, y) lies on the graph if and only if the point (-x, -y) lies on the graph. Here again, once we know
the graph of f on one side of the Y-axis, we know it on both sides.
12
x x0
Example 1.4.3 The function f (x) = x 2 0 x 1
1 x 1
The function whose value at any number x is the greatest integer less than or equal to x is called
the greatest integer function or the integer floor function. It is denoted x observe that
2.4 = 2, 1.9 = 1 0 = 0 -1.2 = -2
2 = 2 0.2 = 0 -0.3 = -1 -2 = -2
13
The graph of the greatest integer function y = x
The function whose value at any number x is the smallest integer greater than or equal to x is called
the least integer function or the integer ceiling function. it is denoted by x
d) f (x) = x3 + x
If f and g are two function with domain D and Dg, then f (x) and g (x) exist for any x that is in
both domain, and we can compute.
f (x) + g (x), f (x) – g (x), and f (x) . g (x).
We can also find f (x) / g (x), provided that g (x) 0
14
The new functions so constructed are called the
sum f + g: x f (x) + g (x),
product f . g: x f (x) . g (x),
difference f – g: x f (x) – g (x),
f f ( x)
quotient :x .
g g ( x)
The domain of f + g, f – g, and f . g is the intersection of the domain of f and of g. the points at
f
which g (x) = 0 must be excluded, however, to obtain the domain of .
g
Example 1.5.1
f (x) = x , g (x) = 1 x
The natural domain of f and g are
D = {x: x 0}, Dg = {x: x 1}
sum f + g: x x + 1 x
difference f – g: x x - 1 x
product f . g: x x 1 x = x(1 x)
f x
quotients :x ,
g 1 x
g 1 x
:x
f x
The domain of f + g, f – g and f . g are all the same [0, 1]. Since g (x) = 0 when x = 1, the domain
f g
of is [0, 1). Similarly, the domain of is (0, 1].
g f
15
1. For each of the following pairs of functions f and g, specify the natural domain of f and g and
f
find the combination f + g, f – g, f . g and and their corresponding domains.
g
1 1
a) f (x) = x, g (x) - 1 x b) f (x) = , g (x) = c) f (x) = 1, g (x) = 1 + x
x2 x 1
1.6 SUMMARY
16
2. a) even b) even c) even d) odd
f–g=x- x 1 = Df – g, fg
fg=x x 1
f x
= Df/g = Df n Dg n g (y) 0
g x 1
g x 1
= Dg/f = Df n Dg n f (y) 0
f x
= [1, +)
1 1
b) f g c) f g 2 x , D f g [1, )
x2 x 1
1 1
f g f g x f g
x2 x 1
1
f g f g 1 x f g
( x 2) x 1
f x 1 f 1
D f / g [1, )
g x2 g 1 x
g x2 g
1 x D g / f [1, )
f x 1 f
Df+g, f – g, f g = Df n Dg
= (1, 2) u (2, +)
Df/g = Df n Dg n g (x) 0 = (1, 2 u (2, +)
Dg/f = (1, 2) u (2, +)
17
1
a) f (x) = 1 + x2 b) f (x) = 1 - x c) f (x) =
x
d) f (x) = 4 x2
2. Graph the functions
a) y = 2x3 b) y = 3 |x|
3. Say whether the function is even, odd, or neither
1
a) f (x) = x3 + x b) f (x) = c) f (y) = 2 |x| + 1
x 1
2
f g
4. Find the domains of f, g and their combination (f g, fg, and )
g f
a) f (x) = x g (x) = x2 + 1
b) f (x) = x 2 4 g (x) = x2
1
c) f (x) = g (x) = 2x
x 1
2
x , x<0
b) Graph the function
f (x) = 1/x , x<0
x x0
18
Unit 2: limits
CONTENT
2.0 Aims and Objectives
2.1 Introduction
2.2 Definition of Limits Using –
2.3 One-Sided Limits
2.4 Finite and Infinite Limits, Vertical, Horizontal Asymptotes
2.5 Limit Theorem
2.5.1 Constant Theorem
2.5.2 Sum Theorem
2.5.3 Difference Theorem
2.5.4 Product Theorem
2.5.5 Quotient Theorem
2.5.6 Chain Rule
2.6 Summary
2.7 Answers to Check Your Progress Questions
2.8 Model Examination Questions
This chapter aims at acquiring students with definition of limits and its application in different area
of applied mathematics; thus at the end of this chapter, you should be able to
define limit using -
find the one sided limit
find the vertical and horizontal asymptotes of functions
state the limit theorems and apply these theorems on different problems
proof limit theorems.
19
2.1 INTRODUCTION
Science seeks to understand the world we live in and mathematics is a primary means to this end.
The laws of nature can be expressed as mathematical equations and manipulated to yield new and
even unsuspected knowledge. Mathematics becomes the “automatic machine” that Frees the mind
from the burden of over whelming details and allows their by the perception of what is really
fundamental.
Calculus in general, limit in particular has many importance. The basic feature of limit process are
easily revealed and do not require any deep mathematical knowledge.
To calculate areas, volumes, velocity, acceleration etc. the concept of limits if Fundamental. So
this chapter tries to define limit and associated applications in it’s sections. There are different
examples and problems that a student should follow smoothly to capture the ideas of limit.
Example 2.2.1 consider the function f (x) = 2x + 3 as x tend to 1. As x tend to 1 from the right
and/or from the left, then f (x) tends to 5, to illustrate.
20
Y
f (x) = 2x + 3
The statement that f (x) tends to 5 as x tends to 1 from right is expressed in symbolic form is
lim f ( x)
=5 and is read
x 1
“The limit of f (x), as x goes to 1 from the right, equals 5” The statement that f (x) tends to 5 as x
tends to a from the left is written as
lim f ( x)
= 5 and is read
x 1
“The limit of f (x), as x goes to 1 from the left, equals 5” The statement that f (x) tends to 5 as x
tends to 1 either from the right or from the left is written
lim f ( x)
= 5 and is read
x 1
x 1
Example 2.2.2 Consider f (x) = as x tends to 1
x 2 1
0
This is an example of rational function that produce indeterminate case (i.e. ) when x is
0
replaced by 1. in this case, it is necessary to cancel the common Factors of the number and the
denominator to determine the appropriate.
21
Limit if it exists, Thus
x 1 x 1
f (x) = =
x 1
2
( x 1) ( x 1)
1
f (x) =
x 1
In order to get the limit as x tends to 1, we used the reduced from to get ½
lim f ( x) lim x 1
Thus =
x 1 x 1 x 2 1
lim lim
=
x 1 x 1
=½
lim f ( x)
Therefore =½
x 1
x 2
Example 2.2.3 consider g (x) = as x tends to 2 again the function g (x) yields
x2 4
0
indeterminate case (i.e. ) when x is replaced by 2, so cancel the common factor of the
0
numerator and the denominator to determine the appropriate limit.
x 2 x 2
g (x) = 2
x 2
x 4
x2
g (x) =
x 2( x 2) ( x 2)
1
g (x) =
( x 2) x 2
lim g ( x) lim 1
x2
=
x 2 x 2 x 2
1 1
=
4 2 2
1 1
=
4 2 2
22
1
=
8 2
lim x 2 1
Therefore =
x2 x 4
2
8 2
lim 2 x 4 lim 25 x 2
b) f)
x 2 x 5
lim 2 x 7 lim x 2 5 x 6
d) h)
x 0 x 3 x3
Definition 2.2.2.1 The limit of f (x) as x goes to a from the right is L, if and only if, for each >
0, there exists some > 0 such that
|f (x) – L| < whenever, a < x < a + .
The statement that the limit of f (x) as x goes to a from the right is L. is expressed by the
equation.
lim f ( x) L
x a
Definition 2.2.2.2 The limit of f (x) as x goes to a from the left is L, if and only if, for each >
0, there exists some > 0 such that
|f (x) – L| < whenever a – < x < a
and it is expressed by the equation
23
lim f ( x) L
x a
Definition 2.2.2.3 The (two-sided) limit of f (x) as x goes to a is L if and only if, for each > 0,
there exists some > 0 such that
|f (x) – L| < , whenever 0 < |x – a| <
lim f ( x) L
The equation is read
xa
“The (two-sided) limit of f (x) as x goes to a equals to L”
lim x 3 8
c)
x 2
P r o o f: -
lim 3x 6
a)
x2
Let > 0, we must find > 0 such that
|f (x) – L| < whenever 0 < |x – a| < f i.e.
|3x – 6| < whenever 0 < |x – 2| < f
But |3x – 6| = |3 (x – 2)|
= /3/ /x – 2/
= 3 /x – 2/
<
Provided that 3 /x – 2/ <
/x – 2/ < /3
Since is small positive number less than 1 (i.e. |x – 2| < 1), we need to define to be the
minimum of 1 and /3, Thus, if we define
24
= {1, /3} then
/3x – 6/ < whenever 0 < /x – 2/ < done
lim 2 x 2a
b)
x 2
Let > 0, we must find > 0 such that
|f (x) – L| < whenever 0 < /x – a/ < i.e.
/2x – 2x/ < whenever 0 < /x – a/ <
But /2x – 2a/ = /2 (x – a)/
= /2/ /x – a/
= 2 /x – a/
<
Provided that 2 / x – a/ <
/x – a/ < /2
Now, define to be the minimum of 1 and /2
Thus, if we define.
= min {1, /2}, then
/2x – 2a/ < whenever 0 < |x – a| <
lim x 3 8
c)
x 2
Let > 0, we must find > 0 such that
/f (x) – L/ < whenever 0 < /x – a/ < i.e.
/x3 – (-8)/ < whenever 0 < /x – (-2)/ <
/x3 + 8/ < whenever 0 < /x + 2/ <
But /x3 + 8/ = /(x + 2) (x2 – 2x + 4/
= /x + 2/ /x2 – 2x + 4/
/x + 2/ (/x2/ + /2x/ + /4/); triangular inequality
/x + 2/ (22 + 2 (2) + 4)
(x + 2/ (4 + 4 + 4)
/x + 2/ (12)
|2 |x + 2|
25
<
Provided that /2/x + 2/ <
/x + 2/ < /12
Since is very small positive number less than 1 (i.e /x + 2/ < 1). We need to define to be the
minimum of 1 and /12. Thus, if we define = min {1, /12} then |x3 + 8| < whenever /x + 2/
< .
lim x 2 9 lim x 4 81
2. 5.
x 3 x 3 x 3 x2 9
lim x2 9
3.
x 3 x 3
Definition 2.3.1.1 Let f be a function defined in some interval to the right of a but not
necessarily at a we say that the right hand side limit of f at a is the real number L.
lim f ( x) L
if and only if as x approaches a but not exactly a, the corresponding value
x a
of f (x) is L.
Example 2.3.1.1: -
Evaluate the following limits.
lim x 1
a)
x 1
26
Solution: - the right hand side limit of x 1 as x approaches to 1 is as follows.
X x 1
1.1 0.1
1.01 0.01
1.001 0.001
1.0001 0.0001
1.00001 0.00001
lim x 1
Therefore =0
x 1
lim x2
b)
x2
x2
x2
Solution: - the right hand side limit of as x approaches to 2 from the right is
x2
x2
X
x2
2.1 0.1/0.1 = 1
2.01 0.01/0.01 = 1
2.001 0.001/0.001 = 1
2.0001 0.0001/0.0001 = 1
2.00001 0.00001/0.00001 = 1
lim x2
Therefore =1
x2
x2
Definition 2.3.2.1 Let f be a function defined in some interval to the left of a but not necessarily
at a. we say that the left hand side limit of f at a is the real number L.
27
lim f ( x) L
if and only if as x tends to a but not exactly at a, the corresponding value of f (x) is
x a
L.
x2
Solution: - the left hand side limit of as x tends to 2 from the left is.
x2
x2
X
x2
1.9 –0.1/0.1 = -1
1.99 –0.01/0.01 = -1
1.999 –0.0001/0.0001 = -1
1.9999 –0.00001/0.00001 = -1
lim x2
Therefore = -1
x2
x2
lim x
b)
x0 x
x
Solution: - the left hand side limit of as x tends to 0 is
x
x
X
x
- 0.1 – 0.1/0.1 = -1
- 0.01 – 0.001/0.001 = -1
- 0.001 – 0.001/0.001 = -1
- 0.0001 – 0.0001/0.0001 = -1
lim x
Therefore = -1
x0 x
28
Remark: -
1. Suppose f is a function and a is a fixed number a real number L is called the limit of f at a
if and only if
lim f ( x) L lim f ( x)
=
xa x a
i.e. the right and the left hand side limits should be equal.
2. The limit of a function f fails to exist
lim f ( x) lim f ( x)
a) if
x a x a
b) either the right of left hand side limit fails to exist.
Solution: - In the above examples the one sided limits are discussed so
lim x lim x
= 1 an d
= -1
x0 x x0 x
lim x lim x
Since = 1 -1 =
x 0 x
x0 x
lim x
Therefore does not exist.
x0 x
lim
b) x 1
x 1
Solution: - Since
lim lim
x 1 = 0 and x 1 does not exist
x 1 x 1
lim
Therefore x 1 does not exist.
x 1
lim x2
c)
x2 x 2
29
lim x2
Solution: - Since = 1 and
x2
x2
lim x2
= -1, thus the right and the left hand side limits are not equal
x2
x2
lim x2
Therefore does not exist.
x2 x 2
lim 1 x 2
d)
x 1 1 x
lim 1 x 2
We have =2
x 1 1 x
30
lim x lim x 2
b) f)
x0
x4 x 4
lim x 1 lim 1
c) g)
x0 x
x2 x 4 2
Definition 2.4.1: - Suppose that a function f is defined on an open interval (a, b) and
a < c< b, then we defined the following limits.
lim f ( x)
a) = + if and only if for every M > 0 there exists some > 0 such that f (x) > M
x c
31
Definition 2.4.2: - Suppose that a function f is defined for all real number where L
lim f ( x) L
i) if and only if for every > 0 there exists some M > 0 such that /f (x) – L| <
x
whenever x > M
lim f ( x) L
ii) if and only if for every > 0 there exists some M > 0 such that /f(x) – L/ <
x
whenever x < -M
lim f ( x) L
iii) = if and only if for every M > 0 there exists some N > 0 such that f(x) > M
x
whenever x > N.
lim f ( x) L
iv) = - if and only if for every M > 0 there exists some N > 0 such that f (x) < -
x
M whenever x > N
lim f ( x) L
v) = if and only if for every M > 0 there exists some N > 0 such that f (x) > M
x
whenever x < - N
lim f ( x) L
vi) = - if and only if for every M > 0 there exists some N > 0 such that f(x) < -M
x
whenever x < -N.
Definition 2.4.3:- The vertical line x = c is called a vertical asymptote to the graph of f if and
only if either
lim f ( x)
a) = or -
x c
lim f ( x)
b) = or - or both
x c
Definition 2.4.4: - The horizontal line Y = L is a horizontal asymptote to the graph of f if and
only if
lim f ( x) L lim f ( x) L
or or both
x x
32
Example 2.4.1 Compute the following limits
lim sin x
a)
x x
Solution: - We know that –1 sin x 1, thus
1 sin x 1
------------- (dividing by x)
x x x
lim 1 lim sin x lim 1
and -
x x x x x x
lim sin x
0 0
x x
lim sin x
Hence =0
x x
sin x
i.e. Y = 0 is the horizontal asymptote of .
x
lim cos x
b)
x x
Solution: - by similar argument in part a
-1 cos x 1
1 cos x 1
-
x x x
lim 1 lim cos x lim 1
-
x x x x x x
lim cos x
0 0
x x
lim cos x cos x
Hence = 0 and Y = 0 is the horizontal asymptote of
x x x
lim x3 2
c)
x 3x 3 2 x 3
Solution: - We divide the numerator and denominator by x3 and then take the limit as follows:
33
x3 2
lim x3 2 lim x3 x3
=
x 3x 3 2 x 3 x 3x 3 2 x 3
x3 x3 x3
2
lim 1
= x3
x 2 3
3
x2 x3
lim 1 lim 2 / x 3
x x
=
lim 3 lim 2 / x 2 lim 3 / x 3
x x x
1 0
=
3 0 0
1
=
3
lim x3 2 1 1 x3 2
Hence = an d y = i s t h e h o ri z o n t al as ym p t o t e o f
x 3x 3 2 x 3 3 3 3x 3 2 x 3
lim x
d)
x2 x 4 2
x
Solution: - To find the right hand side limit of 2 use
x 4
2
x
X
x 4
2
2.1 5.123
2.01 50.125
2.001 500.125
2.0004 5000.125
. .
. .
. .
lim x x
Thus = and x = 2 is the vertical asymptote of 2
x2 x 4 2
x 4
34
lim x
e)
x 1 x 1 2
Solution: - To find the left hand side limit using the above procedure.
x
X
x 1
2
0.9 – 4.737
0.99 – 49.749
0.999 – 499.749
0.9999 – 4999.749
. .
. .
. .
lim x x
Hence = - and x = 1 is the vertical asymptote of 2
using similar argument x
x 1 x 1 2
x 1
x
= -1 is the vertical asymptote of
x 1
2
Find
a) the horizontal Asymptote
b) the vertical Asymptote, for the following functions if possible.
2 x 1 2x
i) f (x) = iv) Y =
3x 2 1 x 2
5x 2 x3
ii) g (x) = v) f (x) =
x3 x2 4
x2 3
iii) h (x) = 2 vi) h (x) = 2 -
x 9 x2
Several of the most useful properties are now set forth, for this purpose consider two functions
each having a limit x a.
Suppose that for some real numbers L and M
35
lim f ( x) L and lim g ( x) M
x a xa
lim k k
where k is a constant function
xa
Proof: - Let f (x) = k, for all x and > 0 be given then using the formal definition of limit
Given > 0, there exists > 0 such that
|f (x) – L| < whenever 0 < |x – a| <
but /f (x) – L/ = /k – k/
=0
<
Choose = , for all x, it holds, this complete the proof.
lim 5 5
Example 2.5.1.1 show that
x 1
Proof: - Given > 0, there exists > 0 such that
/f (x) – L/ < whenever 0 < /x – a/ < i.e.
/f (x) – 5/ < whenever 0 < /x – (-1) / <
but /f (x) – 5/ = /5 – 5/
= /0/
=0
<
Now choose = , ends the proof.
36
lim f ( x) L lim g ( x) M
Proof: - Let > 0 be given and Let and , by definition, there
xa xa
exists 1 > 0 and 2 > 0 such that
/f (x) – L/ < /2 whenever 0 < /x – a/ < 1 --- (1)
/g (x) – M/ < /2 whenever 0 < /x – a/ < 2 --- (2)
Let = min {1, 2}
Then 0 < /x – a/ < implies that
0 < /x – a/ < 1 and /f (x) – L/ < /2 by 1 --- (3)
0 < /x – a/ < 2 and /g (x) – M/ < /2 by 2 --- (4)
Thus if 0 < /x – a/ < , then
/f (x) + g (x) – (L + M) = /f (x) + g (x) – L – M/
= /f (x) – L + g (x) – M/
/f (x) – L/ + /g (x) – M/
< /2 + /2 = by 3 and 4
lim f ( x) g ( x) lim f ( x) lim g ( x)
Therefore
xa xa xa
Solution: -
i) (g + g) (x) = f (x) + g (x), so
lim f ( x) g ( x) lim f ( x) lim g ( x)
x2 x 2 x 2
lim x 2 4 lim 4 x 5
=
x 2 x 2
= 0 + 13
= 13
lim f ( x) g ( x) lim x 2 4 4 x 5
x 2 x 2
37
=
lim x 2 4 4 x 5
x 2
=
lim x 2 4 x 1
x 2
= 13
ii) is exercise
2.5.3 Difference theorem
Now 1 > 0 and by definition of limit there exist 1 > 0 and 2 > 0 such that
/f (x) – L/ < 1 whenever 0 < /x – a/ < 1 --- (5)
/g (x) – M/ < whenever 0 < /x – a/ < 2 --- (6)
Let = min {1, 2}, then 0 < /x – a/ < implies that
0 < /x – a/ < 1 and /f (x) – L/ < 1 by 5 --- (7)
0 < /x – a/ < 2 and /g (x) – M / < 1 by 6 --- (8)
also /f (x) g (x) – LM/ = |(f (x) – L + L) (g (x) – M + M) – LM|
= /(f (x) – L) (g (x) – M) + (f (x) – L) (M) + (L) (g (x) – M) + LM – LM/
= / (f (x) – L) (g (x) – M + M (f (x) –L ) + /g (x) – M)/
/f (x) – L/ /g (x) – M/ + /M (f (x) – L)/ + / L (g (x) – M)/
/f (x) – L/ / g (x) – M/ + /M/ / f (x) – L)/ + L (g (x) – M)/
1 . 1 + /M/ 1 + /L/ 1
38
= 12 + /M/ 1 + /L/ 1
1 + /M/ 1 + /L/ 1 since 1 1 12 1
= (1 + (M) + /L/) 1 taking 1 as a common factor
1 since 1 = 1, this completes the proof.
1 M L
Solution:-
lim f ( x). g ( x) lim f ( x). lim g ( x) by 2.5.4
i)
x 1 x 1 x 1
lim 3x 5 . lim 5 x
=
x 1 x 1
= (2) (-5)
= -10
lim f 2 ( x) lim f ( x) . f ( x)
ii)
x 1 x 1
lim f ( x) . lim f ( x)
=
x 1 x 1
lim 3x 5 . lim 3x 5
=
x 1 x 1
= (2) (2)
=4
2.5.5 Quotient theorem
lim f ( x)
lim f ( x) xa
= , if lim g (x) 0
x a g( x ) lim g ( x)
xa
39
lim
Proof: - Suppose that M > 0 and g (x) = M
xa
1st we need to show that
lim 1 1
x a g ( x) M
1 2
Therefore whenever 0 < /x – a/ < 1
g ( x) M
M2
Let > 0 be given, Let 1 = then
2
Let 1 > 0 and there exists some > 0 such that < 1 and
/ g(x) – M / < 1 whenever 0 < /x – a/ < < 1
1 1 M g ( x)
g ( x) M M g ( x)
M g ( x)
=
M g ( x)
M g ( x)
=
M g ( x)
1
= (/ g (x) – M/ )
M g ( x)
1 2
< . . 1
M M
2 1 M2
= sin ce 1
M2 2
2 M2
=
M2 2
40
= whenever 0 < /x – a/ < .
This completes the proof of the statement
lim 1 1
whenever M > 0
x a g ( x) M
The case for M < 0 can be proved in similar manner. Now we can use theorem 2.5.4 to prove
theorem 2.5.5 as follows:
lim f ( x) lim f ( x) lim 1
= .
x a g ( x) xa x a g ( x)
1
= L.
M
L
=
M
lim f ( x) L
Therefore =
x a g ( x) M
f (x) = x 2 16 , g ( x) x 3 9
Solution: -
lim f ( x)
lim f ( x) x a
i) =
x a g ( x) lim g ( x)
x a
lim x 2 16
lim x 2 16 x 3
=
x 3 x 9
3
lim x 3 9
x 3
5
=
18
lim x 2 16 5
Therefore =
x 3 x 93
18
41
lim g ( x)
lim g ( x) x a
ii) =
x a f ( x) lim f ( x)
x a
lim x 3 9
lim x3 9 x 3
=
x 3 x 2 16 lim x 2 16
x 3
18
=
5
lim x 9
3
18
Therefore =
x 3 x 16
2 5
42
lim gof ( x) Limg ( f ( x)) Lim g ( y )
Therefore
xa xa yL
lim 1 x 2
a)
x 0
lim f ( x)
i) Find
x0
lim f ( x) lim 1 x 2
=
x0 x0
lim f ( x) lim 1 x 2
Thus = =1
x0 x0
lim g f ( x)
ii) Find
x1
lim g f ( x) lim f ( x)
=
x1 x1
=1
lim 1 x 2
Therefore =1
x 0
2.6 SUMMARY
43
lim f ( x) L
a) The statement if and only if
xa
44
lim g ( x)
3. Quotient 0
xa f ( x)
lim f ( x)
Similar properties hold if
xa
Answer 2.2.1
a) 0 d) 7 g) ¼
b) –8 e) 0 h) 1
c) –4 f) 0
45
2.8 MODEL EXAMINATION QUESTIONS
lim 1 lim 1 3 x
5.
x 3 10.
x0 x x 1 x 1
lim f ( x) 3 3 x 2
12. where f (x) =
x2 0 x 2
lim x 0
13.
x 0
lim x 3 0
14.
x 0
lim 3 x 0
15.
x 0
46
1
In exercise 16 – 20 verify the given limits using the formal definition of limits for = . What
100
values of will ensure that / f (x) – L / < whenever 0 < /x – a/ < .
16.
lim 7 x 7
19.
lim 5 2 x 2 3
x 1 x 1
17.
lim x 2 4
20.
lim 4 2 x 3x 2 4
x2 x 0
18.
lim 1 x x 2 1
x 1
In exercise 21 – 23, find the horizontal and vertical asymptotes of the graph of the function.
1
21. f (x) =
x2
2 x x 3
22. g (x) =
9 x2
2x 3
23. h (x) =
4 6x
47
Unit 3: definition of derivative
CONTENT
3.0 Aims and Objectives
3.1 Introduction
3.2 Definition of Derivative
3.3 Tangent and Normal Lines to the Graph of f (x)
3.4 Continuity of Differentiable Functions
3.5 Summary
3.6 Answers to Check Your Progress Questions
3.7 Sample Examination Questions
3.1 INTRODUCTION
The derivative is a limit that measures the rate at which a function changes and is one of the most
important ideas in calculus. In this unit we develop the concept of derivatives.
Definition 3.2.1 Let f be defined on a closed interval [a, b] and a > x < b. then the derivative of f
at x, denoted f (x) , is defined by
lim f x h f ( x)
f (x) = , whenever the limit exists when f (x) exists, we say that f is
ho h
differentiable at x. at the end points a and b, we define one sided derivative as follows
48
lim f x f (a) lim f a h f (a)
i. f (a ) = = we call f (a ) the right-hand
x a xa ho
h
derivative of f at a.
lim f x f (b) lim f b h f (b)
ii. f (b ) = = we call f (b ) the left-hand
xb x
b ho
h
derivative of f at b.
Solution
lim f x h f ( x)
f (x) =
ho h
=
lim x h2 1 ( x 2 1)
ho h
x 2 2hx 1 x 2 1
lim
=
ho h
lim 2hx
= =
ho h
= 2x
f (x) = 2x
lim xhx
=
ho h xh x
lim 1
=
ho xh x
49
1
=
2 x
SOLUTION
lim f x h f ( x)
f (x) =
h0 h
lim sin x h sin x
=
h0 h
lim sin x cos h cos x sin h sin x
=
h0 h
lim sin x cos h 1 cos x sin h
=
h0 h
lim sin x cos h 1 lim
= + cos x sin h
h0 h h0
lim cosh 1 lim
= si n x + cos x si n h
h0 h h0
= sin x (0) + cos x (1)
= cos x
f (x) = cos x
50
3.3 TANGENT AND NORMAL LINES TO THE GRAPH OF F (X)
Recall that a straight line of slope m which passes through the point (x1, y1) is represented by an
equation y – y1 = m (x – x1).
The equation of a line tangent to a given curve y = f (x) at a given point (x1, y1) on the curve can
be found by taking m = f ( x1 ) then it becomes
The line through p1 (x1, y1) and perpendicular to the tangent line is called the normal to the curve
at p. it’s slope is the negative reciprocal of the slope of the tangent. i.e.
1 1 1
m2 = and the equation of the normal line is y – y1 = (x – x1)
m1 f ( x1 ) f ( x1 )
51
= f (a) (0) = 0
lim lim
f (a + h) = f (a) f (x) = f (a)
h0 xa
Hence f is continuous
Note: The converse of theorem 3.4.1 is false is a function need not have a derivative at a point
where it is continuous.
Example 3.4.1 Show that f (x) = |x| is continuous but not differentiable at x = 0
3.5 SUMMARY
f a h f (a)
f (a)
h
The derivative of f at x = a exists if both the left hand and right hand derivatives exists and are
equal at x = a.
A function ƒ differentiable at a point a is continuous at that point and the converse is not true.
52
3.6 ANSWER TO CHECK YOUR PROGRESS QUESTIONS
x x
53
Unit 4: Differentiation Rule
CONTENTS
4.0 Aims and Objectives
4.1 Introduction
4.2 Differentiation Rules
4.3 Implicit Differentiation
4.4 Differentials
4.5 Higher Derivatives
4.6 Summary
4.7 Model Examination Questions
4.8 Answer to Check Your Progress Questions
4.1 INTRODUCTION
When we can not write an equation of the form F(x, y) = 0 in the form of y = f(x) it is not possible
to differentiate it in the usual way. In this unit we will introduce the concept of implicit
differentiation to solve such problems.
Theorem 4.2.1. Suppose that function f and g are defined on some open interval (a, b) and f1 (x)
and g1 (x) exists at each point x in (a, b) then
54
i. (f + g) (x) = f (x) + g(x) ---------(The sum rule)
ii. (f – g) (x) = f (x) - g (x) ----------(The difference rule)
iii. (kf) (x) = k f (x) , k R (The constant multiple rule)
iv. (f . g) (x) = f (x) g(x) + g (x) f(x). ----------(The product rule)
f ( x) g ( x) f ( x) g ( y )
v. f g (x) = , g(x) 0 (The Quotient rule)
g ( x)2
Proof
lim [ f ( x h) g ( x h)] [ f ( x) g ( x)]
i. (f + g) (x) =
h0 h
lim f ( x h) f ( x) lim g ( x h) g ( x )
= +
h0 h h0 h
= f (x) + g (x)
lim [ f ( x h) g ( x h)] [ f ( x) g ( x)]
ii. (f – g) (x) =
h0 h
lim f ( x h) f ( x) lim g ( x h) g ( x )
= -
h0 h h0 h
= f (x) + g (x)
lim kf ( x h) kf ( x)
iii. (kf) (x) =
h0 h
lim f ( x h) f ( x )
=k.
h0 h
= k f (x)
lim [ f ( x h) g ( x h)] [ f ( x) g ( x)]
iv. (f . g) (x) =
h0 h
lim 1
= [(f(x + h) – f(x)) g(x + h) + f(x) (g(x + h) – g(x))]
h0 h
lim f ( x h) f ( x) lim lim ( g ( x h) g ( x))
= . g(x + h) + f(x)
h0 h h0 h0 h
= f (x) g(x) + f(x) g (x)
55
f lim 1 f ( x h) f ( x )
v. (x) =
g h 0 h g ( x h) g ( x )
lim 1 f ( x h).g ( x) g ( x h) f ( x)
=
h0 h g ( x h) g ( x )
1 lim f ( x h) f ( x ) ( g ( x h) g ( x))
= g ( x) f ( x)
( g ( x)) h 0
2
h h
1
= [f/(x) g(x) – f(x) g/(x)]
( g ( x)) 2
g ( x) f ( x) g ( x) f ( x)
= , if g(x) 0
( g ( x)) 2
Note: To emphasize the fact that the derivatives are taken with respect to the independent variable
x, we use the following notation
d
f/ (x) = (f(x))
dx
Theorem 4.2.2
d (k )
i. = 0, for k R
dx
d n
ii. (x ) = nxn-1, for each real number x and natural number n.
dx
d
iii. (Sin x) = Cos x, x R
dx
d
iv. (Cos x) = -Sin x, x R
dx
d
v. (tan x) = Sec2x, x R x (2n + 1) , n Z
dx 2
d
vi. (cot x) = -csc2x. x R x n, n Z
dx
d
vii. (Sec x0 = Sec x tan x, x R x (2n + 1) , n Z
dx 2
d
viii. (Csc x) = -csc x cot x, x R x n, n Z
dx
56
Pro o f
d lim k k
i. (k) =
dx h0 h
lim 0
=
h0 h
=0
ii. For each natural number n, we get
d n lim ( x h) n x n
(x ) = using binomial expansion
dx h0 h
lim 1 n n 1 n(n 1) n 2 2 n
x nx h 2! x h h x
n
=
h0 h
lim n1 n(n 1) n 2
= nx x h h n 1
h0 2!
= nxn-1
d d Sin x
v. (tan x) =
dx dx Cos x
57
ii. 4 sin x – 3 cos x
d d d
Solution. ( 4 sin x – 3 cos x) = 4 (sin x) - 3 (cos x)
dx dx dx
= 4 cos x – 3 (-sin x)
= 4 cos x + 3 sin x
iii. x sin x + x2cos x
Solution. Using the sum and product rule, we get
d d d 2
(x sin x + xn cos x) = (x sin x) + (x cos x) (Sum rule)
dx dx dx
d d d 2 d
= [ si n x (x) + x (sin x)] + [cos x (x ) + x2 (cos x)]
dx dx dx dx
= sin x + x cos x + 2x cos x + x2 (-sin x)
= sin x + 3 x cos x – x2 sin x
x3 1
iv.
x2 4
Solution. Using the sum and quotient rule, we get
d x 3 1 x 2 4 dxd x 3 1 x 3 1 dxd x 2 4
2 =
dx x 4 x 2 42
( x 2 4)(3x 2 ) ( x 3 1)(2 x)
=
( x 2 4) 2
3x 4 12 x 2 2 x 4 2 x
=
( x 2 4) 2
x 4 12 x 2 2 x
=
( x 2 4) 2
58
THE CHAIN RULE
Suppose we have two functions, u and y, related by the equations u = g(x) and y = f(u)
Then y = (fog) (x) = f(g(x)).
The chain rule deals with the derivative of the composition and may be stated as the following
theorem:
Theorem 4.2.3. (The chain rule). Suppose that g is defined in an open interval I containing c, and
f is defined in an open interval J containing g(c), such that g(x) is in J for all x in I. If g is
differentiable at c, and f is differentiable at g(c), then the composition (fog) is differentiable at c
and
(fog)/ (c) = f/ (g(c)) . g/ (c)
In general, if u = g(x) and y = f(u), then
dy dy du
.
dx du dx
Proof. Let F be defined on J such that
f (u ) f ( g (c))
if u g (c)
F(u) = u g (c)
f ( g (c)) if u g (c)
Since f is differentiable at g(c),
lim F (u ) lim f (u ) f ( g (c))
=
u g (c ) u g (c ) u g (c )
= f/(g(c))
= F(g(c))
Therefore, F is continuous at g(c). By the definition of F, f(u) – f(g(c)) = F(u) (u – g(c)) for all u
in J. For each x in I, we let y = g(x) on I and
lim ( fog )( x) ( fog )c
(fog)/(c) =
xc xc
lim f ( g ( x)) f ( g (c)) g ( x) g (c)
= .
xc g ( x ) g (c ) xc
lim F (u ) lim g ( x ) g (c )
= .
u g (c ) x c xc
= f/(g(c)) . g/(c)
59
Example 4.2.2. Find the derivative of y = x2 1
Solution. Here y = f(g(x)) where f(u) = u and g(x) = x2 + 1. Since the derivative of f and g are
1
f/(u) = and g/(x) = 2x
2 u
the chain rule gives
dy d
f(g(x)) = f/ (g(x)) . g/(x)
dx dx
1 1
= . g/(x) = . (2x)
2 g ( x) 2 x2 1
x
=
x 1
2
60
CHECK YOUR PROGRESS 4.2.3
Compute the derivative of the given function
3
x3 1
1. y = 2 2. y = (x2 – 1)10 3. (2 sin – 3 cos x2)3
x 4
4. Sin (3x + 2) 5. Cos (3x2+1) 6. y = x sin
1
x2
61
dy
Example 4.3.1. Find if x2 + y2 = 16
dx
Solution. Assuming that y is to be considered as a function of x, we differentiate each term of the
equation with respect to x.
d 2 d d
(x ) + (y2) = (16)
dx dx dx
dy
2x + 2y =0
dx
dy
2y = -2x
dx
dy x
= ,y0
dx y
dy x
: . y/ = =
dx y
dy
Example 4.3.2. Compute for the equation x3 + y3 = 4xy
dx
Solution. As we do in above we consider the same assumption
d 3 d 3 d
(x ) + (y ) = (4 x y)
dx dx dx
dy d d
3x2 + 3y2 = 4[ (x)y + x (y)]
dx dx dx
dy dy
3x2 +3y2 -4x = 4y
dx dx
dy dy
3y2 -4x = 4y – 3x2
dx dx
dy
(3y2- 4x) = 4y – 3x2
dx
dy 4 y 3x 2
= , if 3x2 – 4x 0
dx 3y 2 4x
4 y 3x 2
/
:.y =
3y 2 4x
62
dy
Example 4.3.3. Find if 2y = x2 + sin y
dx
Solution. 2y = x2 + sin y
d d 2
(2y) = (x + sin y)
dx dx
dy dy
2 = 2x + cos y
dx dx
dy dy
2 - cos y = 2x
dx dx
dy
(2 – cos y) = 2x
dx
dy 2x
=
dx (2 cos y )
Example 4.3.4. Find the tangent and normal to the curve x2 – xy + y2 = 7 at the point
(-1, 2)
dy
Solution: We first use implicit differentiation to find
dx
d 2 d
(x – xy + y2) = (7)
dx dx
d 2 d d 2
(x ) - (xy) + (y ) = 0
dx dx dx
dy dy
2x – y - x + 2y =0
dx dx
dy
(-x + 2y) = y – 2x
dx
dy y 2 x
=
dx 2 y x
We then evaluate the derivative at (x, y) = (-1, 2) to obtain
dy y 2x 2 2(1) 4
1, 2 = 1, 2 =
dx 2y x 2(2) (1) 5
63
The tangent to the curve at (-1, 2) is the line
4
y=2+ (x-(-1))
5
4 14
y= x+
5 5
The normal to the curve at (-1, 2) is
5
y=2- (x-(-1))
4
5 3
y=- x+
4 4
5. x y = xy - x
II. Find the equation of the tangent and normal to the graph of the given equation at the given
point
1. x2 + xy – y2 = 1, (2, 3) 2. y = 2 sin (x – y), (1, 0)
4.4 DIFFERENTIALS
Definitions
Let y = f(x) be a differentiable function. The differential dx is an independent variable. The
differential dy is
dy = f/(x) dx
Example 4.4.1 Find dy if
a) y = x5 + 37x b) y = sin 3x
64
Solution
a) dy = (5x4 + 37)dx b) dy = (cos 3x) . 3 dx
= 3 cos 3x dx
If dx 0 and we divide both sides of the equation dy = f/(x)dx by dx, we obtain
dy /
= f (x)
dx
1. y = x3- 3 x
2. y = sin 5 x 3. y = x 1 x 2
If the vertical height y of an object is a function f of time, then y/(f) is called its velocity; denoted
v(t). The derivative v/(t) is called the acceleration of the object and is denoted a(t). i.e.,
y(t) = f(t), y/(t) = v(t), v/(t) = a(t)
We say that a(t) is the second derivative of y, with respect to t, and write
// d2y
y (t) = a(t) or = a(t)
dt 2
Derivatives of order two or more are called higher derivatives and are represented by the following
notation:
dy // d 2 y /// d3y dny
y/(x) = , y (x) = , y (x )= , ---, y(n)
(x ) =
dx dt 2 dx 5 dx n
The definition is given as follows by induction:
d2 f d df d2 f d d n 1 f
= an d = , n = 2, 3, 4, ---
dx 2 dx dx dx 2 dx dx n 1
d2 f
A convenient notation is f(n)(x) =
dx 2
Which is read as “the nth derivative of f with respect to x.”
Example 4.5.1. Compute the second derivative y// for each of the following function:
65
1. y = sin(3x) 2. y = cos (4x2)
Solution. y/ = cos 3x . (3x)/ Solution. y/ = -sin (4x2) . (4x2)/
= 3 cos 3x = -8x sin 4x2
y// = -3 sin 3x . 3 y// = (-8x)/ sin 4x2 + (-8x) (sin 4x2)/
= -9 sin 3x = -8 sin 4x2 + (-8x) cos 4x2 . 8x
= -8 sin 4x2 – 64x2 cos 4x2
= -8 (sin (4x2) + 8x2 cos (4x2))
3. f(x) = 2x3 – 6x 4. y = x2 sin x
Solution. f/(x) = 6x2 – 6 y/ = (x2)/ sin x + x2(sin x)/
f//(x) = 12x = 2x sin x + x2 cos x
y// = (2x)/ sin x + 2x(sin x)/ + (x2)/ cos x + x2(cos x)/
= 2 sin x + 2x cos x + 2x cos x – x2sin x
4.6 SUMMARY
- Differentiation rules
Let f and g are differentiable functions defined on an open interval (a, b).
1. (f g)/ = f/ g/
2. (kf)/ = kf/, k R
3. (f . g)/ = f/g/
4. f g f g
- (k)/ = 0, k R
(xn)/ = nxn-1, n N and x R
66
(sin x)/ = cos x (tan x)/ = sec2x
(cos x)/ = -sin x
- The chain rule
The derivative of the composite function f(g(x)) at x is the derivative of f at g(x) times the
derivative of g at x. i.e., (fog(x))/ = f/ (g(x)) . g/(x)
- Derivative of order two or more called higher derivatives.
x 3 1 x
4
3 2
d) (1 + ) e) f)
x x 3 1 x
g) sin 3x h) sin 1 x 2
x
2. Suppose that f(x) is a differentiable function and that f/(x) = , Find the derivative of
1 x
each of the following function:
3 x
a) f(x + 2) b) f(x2) c) f( x ) d) f
2 x
3. Find the indicated derivative
1
a) f//(x) for f(x) = b) f//(x) for f(x) = x si n x
x2
d4y d3y
c) for y = (x3-1)4 d) for y = x4/3 + x1/4
dx 4 dx 3
dy
4. Find by implicit differentiation
dx
a) x2 + y2 = 25 b) x2y + xy2 = 0 c) x 4 = xy – x
x 1
d) cot y = 3x2 + cot (x+y) e) y2 =
x2 1
5. Find the equation of the line tangent to the graph determined by the given equation at a given
point and also find the equation of the normal line at this point.
67
a) xy = 9, (3, 3) b) x2 + y2 = 4, ( 2 , 2 )
x y
c) = 4, (5, 3) d) (y – x)2 = x, (9, 12)
x y
6x 2 x3
4. Sec x (5 tan x – 3 sec x) 5.
(3x 1) 2
6. 2x tan x + x2 sec2x + cot x – x csc2x
3x 2 2 xy y 2 y2 2( y 1) x y 1
4.
x 2 2 xy 3 y 2
5.
y sin
1
y cos xy
1
y
6.
1 2x x y
II.
68
7 1
1. The equation of the tangent line is: y = x-
4 2
4 29
the equation of the normal line is: y = x+
7 7
2. tangent line: y = 2x - 2
x 1
normal line: y = -
2 2
69
Unit 5: application of derivatives
CONTENTS
5.0 Aims and Objectives
5.1 Introduction
5.2 Extrema of a Function, Critical Number of a Function. Roles Theorem and Mean Value
Theorem
5.3 Monotonic Functions, Relative Extrema
5.4 Concavity: Point of Inflection, Second Derivative of the Signs of the First and Second
Derivatives of f (x) in regard to the Graph of f (x)
5.5 Application of the Theory of Extrema
5.6 Velocity and Acceleration
5.7 Summary
5.8 Answers to Check Your Progress Questions
5.9 Model Examination Questions
In this chapter we continue the discussion of properties of the derivatives of functions. We will be
principally concerned with simplifying the process of a differentiation so that it will longer be
necessary to deal with complicated limits. Thus at the end of this chapter an individual should be
able to:
derivate a function
re-write definition of extrema
locate the extrema of a function
re-write role’s theorem, mean value theorem
apply roles and mean value theorems to find the critical number, local extrema of a
function
find an interval where the given function is increasing or decreasing
use second derivative test to identify the relative extrema of a function.
70
5.1 INTRODUCTION
The development of calculus grew out from four major problems, these problems are
i) the tangent line problem
ii) the velocity and acceleration problem
iii) the minimum and maximum problem
iv) the area problem
Each problem involves the notion of a limit and we could introduce calculus with any one of the
four problems. In the previous until tangent line problem is solved. So in this unit, we will
introduce, How to find velocity and acceleration of moving objects. How to calculate the minimum
and maximum of a function. for this purpose we have to look at extrema of functions monotonicity,
concavity. First and second derivative test and finally we will introduce application problems.
One of the most important problems in the real world is optimization. This is the problem of
maximizing or minimizing a given function. Differentiation plays a key role in solving such real
world problems.
Definition 5.2.2: A function f with domain I is said to have a minimum at c if f (c) f(x) for all
x I.
The number f (c) is called the local minimum (relative minimum) of f if there is some open interval
(a, b) containing c and f (c) is the minimum of f on (a, b).
Definition 5.2.3: The minimum or the maximum value of f is called the extreme value or extrema
of the function on the interval.
71
Remark: - The maximum and minimum values of a function on an interval are sometimes called
the absolute minimum and absolute maximum respectively.
x 3 3x 2
Example 5.2.1: - Let us use the graph of f (x) = to locate the extrema of f on the open
2
intervals
a) (-2, 4) b) (–2, 3) c) (–1, 3) d) (–1, 4)
DEFINITION 5.2.4
1. If there is an open interval in which f (c) is a maximum then f (c) is called a relative
maximum of f
2. If there is an open interval in which f (c) is a minimum then f (c) is called a relative
minimum of f.
72
Example 5.2.2 In the following figure, the points (0, 0) and (2, -2) are relative maximum and
minimum values respectively.
If f is continuous on a closed interval, then f has both a minimum and maximum values in the
intervals.
Example 5.2.3 the function f (x) = x – x3 for 0 x 1 is continuous on a closed interval [0, 1] by
theorem 5.2.1; f has a maximum and minimum values on [0, 1]. But the theorem does not indicate
at what point is maximum and minimum values. So the next theorem will illustrate.
Theorem 5.2.2 If f has a relative minimum or relative maximum at x = c, then c is a critical number
of f
PROOF: -
Case I: - If f is not differentiable at x = c, then by definition c is a critical number of f and we have
nothing to prove.
73
Case II: - If f is differentiable at x = c, then f (c) must be either positive, negative or zero. Suppose
f (c) is negative, then we have
lim f ( x ) f (c )
f (c) = 0
xc xc
this implies that there exists an interval (a, b) containing c such that
f ( x ) f (c )
0 for all x c, c (a, b) since the quotient is negative, the sign of the
xc
denominator and numerator must be opposite (i.e. one is positive and the other is negative). Now
let us make an assumption for all values of x in the interval.
Assuming that f (c) > 0 produces similar, contradiction the only possibility is f (c) = 0
Remark: - To find the extrema of a continuous function f on a closed interval [a, b], we suggest
the following steps.
a) Evaluate f at each of its critical number in (a, b)
b) Evaluate f at each end points of [a, b]
c) The least of these values is the minimum and the largest is the maximum
EXAMPLE 5.2.4
I. Find all critical points (if any) of the given function
a) f (x) = 3x4 – 4x3
74
f (x) = 12x3 – 12x2
12x3 – 12x2 = 0
12x2 (x – 1) = 0
12x2 = 0 V x –112 = 0
x=0Vx=1
The critical points are 0 and 1
b) g (x) = 4x3 – 6x2 – 9x
Solution: - To find the critical point use g (x) = 0
g (x) = 12x2 – 12x – 9
12x2 – 12x – 9 = 0
(2x – 3) (6x + 3) = 0
2x – 3 = 0 V 6x + 3 = 0
2x = 3 V 6x = -3
x = 3/2 V x = -3/6 = -1/2
The critical points are –1/2 and 3/2
c) h (x) = x3 – 12x
Solution: - To find the critical points use h (x) = 0
h (x) = 3x2 – 12
3x2 – 12 = 0
3 (x2 – 4) = 0
3 (x + 2) (x – 2) = 0
x+2=0Vx–2=0
x = -2 V = x = 2
The critical points are –2 and 2.
1
d) f (x) = x +
x
75
Solution: - To find the critical points use f (x) = 0
1
f (x) = 1 -
x2
1
1- =0
x2
x 2 1
=0
x2
x2 – 1 = 0
(x – 1) (x + 1) = 0
x–1=0Vx+1=0
x = 1 V x = -1
Therefore the critical points are –1 and 1
II. Find all extreme values (if any) of the given function on the given interval. Determine at which
numbers in the interval these values occurs.
a) f (x) = x2 – x on [0, 2]
Solution: - To find the extrema of a function we use the remark.
f (x) = x2 – x
f (x) = 2x – 1
2x – 1 = 0
2x = 1
x = ½ [0, 2]
the critical point is ½
interval (i.e. x = 2)
b) g (x) = x2 – 5x – 6 on [5/2, 6]
Solution: - To find the extrema of the function use g (x) = 0
g (x) = 2x – 5
76
2x – 5 = 0
2x = 5
x = 5/2 [5/2, 6]
The critical point is 5/2
end point critical point end point
f (5/2) = -12.25 f (5/2) = -12.25 f (6) = 0
Thus the relative minimum of the function is –12.25 at a point 5/2 and the relative maximum
value of the function is 0 at a point 6.
Solution: - Note that f is differentiable on the entire real line. Setting f (x) equal to zero,
we have f (x) = x2 – 3x + 2
x2 – 3x + 2 = 0
(x – 1) (x – 2) = 0
77
Thus f (1) = f (2) = 0 and from Rolle’s theorem we know that there exists c in the interval
(1, 2) such that f (c) = 0. to find c, we solve the equation f (x) = 0 as follows:
f (x) = 2x – 3
2x – 3 = 0
2x = 3
x = 3/2 (1, 2)
b) Let f (x) = x4 – 2x2, Find all c in the interval (-2, 2) such that f (c) = 0
Solution: - Since f (-2) = 8 = f (2) and f is differentiable Rolles theorem guarantees the
existence of at least one c in (-2, 2) such that f (c) = 0. Setting the derivative equal to
zero, we have
f (x) = x4 – 2x2
f (x) = 4x3 – 4x
f (x) = 4x (x2 – 1)
4x (x2 – 1) = 0
4x = 0 V x2 – 1 = 0
x = 0 V (x – 1) (x +1) = 0
x=0vx–1=0Vx+1=0
x = 0 V x = 1 V x = -1
If f is continuous on [a, b] and differentiable on (a, b), then there exists a number c in
f (b) f (a)
(a, b) such that f (c) =
ba
Proof: - In the figure below, the equation of scant line continuing the points (a, f (a)) and (b, f (b))
is given by
78
f (b) f (a)
Y= x a f (a)
ba
Let g (x) be the difference between f (x) and Y then
f (b) f (a)
g (x) = f (x) – Y = f (x) - x a f (a)
ba
Evaluating the function g at a and b, we have
f (b) f (a)
g (x) = f (x) - x a f (a)
ba
f (b) f (a)
g (a) = f (a) - a a f (a)
ba
= f (a) – 0 – f (a)
= f (a) – f (a)
=0
g (a) = 0
f (b) f (a)
g (b) = f (b) - b a f (a)
ba
g (b) = f (b) – f (b) f (a) - f (a)
g (b) = f (b) – f (b) + f (a) – f (a)
g (b) = 0 + 0
g (b) = 0
Thus g (a) = 0 = g (b)
and since f is differentiable, g is differentiable and we can apply Rolle’s theorem to the function g
there exists a point c in (a, b) such that g (c) = 0 this means that
f (b) f (a)
0 = g (c) = f (c) -
ba
79
f (b) f (a)
0 = f (c) -
ba
f (b) f (a)
f (c) =
ba
f (b) f (a)
Therefore f (c) =
ba
Remark: - The “mean” in the mean value theorem refers to the mean (or average) rate of change
of f in the interval [a, b]
EXAMPLE 5.2.6
4
I) Given f (x) = 5 , find all c in the interval (1, 4) such that
x
f (4) f (1)
f (c) =
4 1
Solution: - The slope of the scant line through (1, f (1)) and, (4, f (4)) is
f (4) f (1) 4 1
= =1
4 1 4 1
Since f satisfies the condition of the mean value theorem there exists at least one c in
(1, 4) such that
f (c) = 1 solving the equation f (x) = 1, we have
f (x) = 4/x2 = 1
4 = x2
x2 – 4 = 0
(x – 2) (x + 2) = 0
x–2=0Vx+2=0
x = 2 V x = -2
2 (1, 4) and –2 (1, 4), we choose c = 2
II) Find the number c that satisfies the conclusion of the mean value theorem for the given function
and interval.
a) f (x) = 1 + 2x2 on [-1, 1]
Solution: - given a = -1 and b = 1
80
f (b) f (a)
f (c) =
ba
f (1) f (1)
f (c) =
1 (1)
33 0
f (c) = 0
11 2
f (c) = 0
Since f satisfies the condition of the mean value theorem there exists at least one c in
(-1, 1) such that f (c) = 0 solving the equation
f (x) = 4x = 0
4x = 0
x = 0/4
x=0
Since 0 (-1, 1), we choose c = 0
b) g (x) = x3 – x on [-1, 1]
Solution: - given a = -1 and b = 1
g (b) g (a)
g (c) =
ba
g (1) g (1)
g (c) =
1 (1)
00 0
g (c) = 0
11 2
Since g satisfies the condition of mean value theorem there exists at least one c in (-1, 1) such that
f (c) = 0 solving the equation f (x) = 0, we have
f (x) = 3x2 – 1 = 0
3x2 – 1 = 0
3x2 + 1
1 3
x=
3 3
81
3 3 3
Since (-1, 1) and (-1, 1), we choose c = .
3 3 3
c) A Landowner wishes to use 4 miles of fencing to enclose a rectangular region of maximum area
what should be the length of the sides be?
Solution: - the area of any rectangular region is given by A = bh (base x height) and the
perimeter is p = 2b + 2h = 4miles
2b + 2h = 4miles
2 (b + h) = 4miles
b + h = 2 miles for 0 b 2 and 0 h 2 and b = 2 – h, so the area of
rectangular region is
A = bh
A = (2 – h) h
A = 2h - h2 for 0 h 2
So to find the maximum value of the area A on [0, 2] we use mean value theorem
A (h) = 2h – h2
A (h) = 2 – 2h
2 – 2h = 0
-2h = -2
h=1
The maximum value of the area occurs at h = 1 mile so the rectangular region should have base b
= 1 mile and height h = 1 mile to obtain maximum area of 1 sq. mile.
82
1) f (x) = x (x2 – x – 2)
2) f (x) = /x/ - 1
3) f (x) = x + sin x
4) f (x) = x2 – 3x + 2
5) f (x) = (x + 3) (x - 1)2
In exercise 6 – 10 apply the mean value theorem on the indicate interval and find all values of
f (b) f (a)
c in the interval (a, b) such that f (c) =
ba
6) g (x) = x2 on [-2, 1]
7) g (x) = x/x+1 on [-1/2, 2]
8) g (x) = x – 2sin x on [- ]
9) g (x) = x 2/3 on [0, 1]
10) g (x) = x3 – 12x + 4 on [-3, 3].
In the previous sections wee have seen how derivatives are useful in locating the relative extrema
of a function. In this section, we will see how to classify a function as either relative mnima or
relative maxima using the concept of derivative. For this purpose let us define increasing and
decreasing functions.
Definition 4.3.1
1. A function f is side to be increasing on an interval if for any two numbers x1 and x2 in the
interval x1 < x2 implies f (x1) < f (x2)
2. A function f is said to be decreasing on an interval if for any two numbers x1 and x2 in the
interval x1 < x2 implies f (x1) > f (x2)
Example 5.3.1: - Categorize the function as increasing or decreasing function if possible on the
given interval
a) f (x) = 3x + 5 on (- )
83
Solution: - Let x1, x2 (-, ) and x1 < x2
f (x1) = 3x1 + 5
f (x2) = 3x2 + 5
Since x1 < x2 3x1 < 3x2 (multiplication property)
3x1 + 5 < 3x2 + 5 (addition property)
f (x1) < f (x2)
Since x1 < x2 f (x1) < f (x2)
f is an increasing function
b) g (x) = -x + 7 on (- )
Solution: - Let x1, x2 (-, ) and x1 < x2
but g (x1) = -x1 + 7
g (x2) = -x2 + 7
Since x1 < x2 -x1 > -x2 (property of multiplication)
-x1 + 7 > -x2 + 7 (addition property
g (x1) > g (x2)
Since x1 < x2 g (x1) > g (x2)
g is decreasing function
84
2. If f (x) < 0, for all x in (a, b), then f is decreasing on (a, b)
3. If f (x) = 0, for all x in (a, b), then f is constant on (a, b)
Proof: - To prove the 1st case, we assume that f (x) > 0, for all x on (a, b) and Let x1<x2 be any
two points in the interval. By mean value theorem, there exists a number c such that x1 < c < x2
and
f ( x 2 ) f ( x1 )
f (c) =
x 2 x1
Since f (c) > 0 and x2 – x1 > 0, we conclude that f (x2) – f (x1) > 0 which implies f (x2) > f (x1).
Therefore x1 < x2 f (x1) < f (x2), thus f is increasing on the interval. The other cases can be
proved similarly.
Example 5.3.2: - For the following functions find an interval for which the function is increasing
or decreasing
a) f (x) = x3 – 3/2x2
Solution: - Find the first derivative of the function
f (x) = x3 – 3/2x2
f (x) = 3x2 – 3x = 0
3x2 – 3x = 0
3x (x – 1) = 0
3x = 0 V x – 1 = 0
x = 0 V x = 1 are critical numbers
Thus
Interval -<x<0 0<x<1 1<x<
Test value x = -1 x=½ x=2
85
Sing of f (x) f (1) = 6 > 0 f (1 / 2) = -3/4 < 0 f (2) = 6 > 0
Conclusion Increasing Decreasing Increasing
2 1
2 x
f (x) =
3 2 1/ 3
x 4
4x
f (x) = 0
x 2
4
1/ 3
4x
0
x 2
4
1/ 3
4x = 0
x = 0 and x = 2 is undefined for f (x) thus x = 0, x = 2 are critical numbers.
86
x 4 1
c) f (x) =
x2
x 4 1
Solution: - Find the derivative of the function f (x) =
x2
x 2 4 x 3 2 x x 4 1
f (x) =
x2
4x 5 2x 5 2x
f (x) =
x4
2x5 2x
f (x) =
x4
2 x x 4 1
f (x) =
x4
2 x 4 1
f (x) =
x3
2 x 2 1x 2 1
f (x) =
x3
=
2 x 1x 1 x 2 1
x3
since f (x) is zero at x = 1 and f is discontinuous at x = 0 we use three values to determine the
test intervals.
x = -1 and x = 1 an critical numbers
x = 0 is a point of discontinuity
Interval - < x < -1 –1 < x < 0 0<x<1 1<x<
Test value x = -2 x = -1/2 x=½ x=2
Sign of f (x) f (2) < 0 f (1 / 2) > 0 f (1 / 2) < 0 f (2) > 0
Conclusion Decreasing Increasing Decreasing Increasing
87
2. If f (x) changes from positive to negative at c, then f (c) is a relative maximum of f
3. If f (x) does not change signs at c, then f (c) is neither a relative minimum nor a relative
maximum.
Example 5.3.4 Use the First Derivative test to find all relative maxima and minima for the
following functions.
a) f (x) = x/2 – sin x on [0, 2]
Solution: -
f (x) = x/2 – sin x
f (x) = ½ - cos x = 0
From the above, we conclude that a relative minimum occurs at x = /3 and a relative maximum
occurs at 5/3.
88
Remark: - If a function f is either increasing or decreasing on the interval D, then f is said to be
monotonic on D. f is said to be strictly monotonic on D, if it is either strictly increasing or strictly
decreasing on D.
1. The electrical power p in watts in a direct current circuit with two registers R1 and R2 connected
in series is
VR1 R2
P=
R1 R2 2
Where V is the voltage. If V and R1 are held constant. What resistance R2 produces maximum
power?
2. After birth, an infant will normally lose weight for few days and then start gaining. A model for
the average weight W of infants over the first two weeks following birth is
W = 0.33t2 – 0.3974t + 7.3032
Find the interval on which W is increasing or decreasing.
3. The height (in feet) of a ball at a time t (in second) is given by the position function
s (t) = 96t – 16t2
Find the time interval on which the ball is moving up and the interval on which it is moving
down. What is the maximum height of the ball?
4. Coughing forces the trachea (wind pipe) to contract, which in turn affects the velocity V of the
air through the trachea. Suppose the velocity of the air during coughing is
V = k (R – r) r2
Where k is a constant R is the normal radius of the trachea and r is the radius during coughing.
What radius will produce the maximum air velocity?
5.4 CONCAVITY AND SECOND DERIVATIVE TEST
In the previous section, we have seen how to locate the interval in which a function f increases or
decreases. Here we will see how the function is curving upward or down ward, we define this
notion of curving upward or downward as concavity.
89
Definition 5.4.1: - Let f be differentiable on the open interval. We say that the graph of f is concave
upward if f is increasing on the interval and concave downward if f is decreasing on the
interval.
Remark: -
1. If a curve lies above its tangent lines, then it is concave upward.
2. If a curve lies below its tangent lines, then it is concave downward.
Example: - 5.4.1. Determine the interval in which the following functions are concave upward or
downward
6
a) f (x) =
x 3
2
90
12 x
f (x) =
( x 2 3) 2
f (x) =
x 2
12 12 x 2x
3
2 2
3 2 x
x 3 2 4
12 x 3 48 x x 3
2 2 2 2
f (x) =
x 3 2 4
12 x 3 48 x2 2
f (x) =
x 3 2 3
12 x 2 36 48 x 2
f (x) =
x 2
3
3
36 x 2 36
f (x) =
x 3 2 3
36 x 1 2
f (x) =
x 3 2 3
Now f (x) = 0 for x = 1 and f (x) is defined on the every real number. We test f in the
following intervals.
Interval - < x < -1 –1 < x < 1 1<x<
Test value x = -2 x=0 x=2
Sign of f (x) f (-2) > 0 f (0) < 0 f (2) > 0
Conclusion concave concave concave
upward downward upward.
x 2 1
b) f (x) =
x2 4
Solution: - We find the second derivative
x 2 1
f (x) =
x2 4
91
f (x) =
x 2
4 2 x x 2 1 2 x
( x 2 4) 2
2 x 3 8x 2 x 3 2 x
f (x) =
( x 2 4) 2
10 x
f (x) =
( x 2 4) 2
f (x) =
10 x 2 4 10 x 2 x 2 4 2 x
2
( x 2 4) 4
f (x) =
2
10 x 2 4 40 x 2 x 2 4
( x 2 4) 4
f (x) =
10 x 2 4 40 x 2
2
( x 2 4) 3
10 x 2 40 x 2 40
f (x) =
( x 2 4) 3
30 x 2 40
f (x) =
( x 2 4) 3
f (x) =
10 3x 2 4
( x 2 4) 3
There is no point that f (x) is zero. But at x = 2 the function f is discontinuous. So we test for
92
Definition 5.4.2. If the graph of a continuous function possesses a tangent line at a point where its
concavity changes from upward to downward (vice versa). We call the point a point of inflection.
Since a point of inflection occurs where concavity of a graph changes, it must be true that the sign
of f changes at such points.
Thus to locate possible point of inflection we need only determine the value of x, for which f
(x) = 0 or for which f (x) is undefined
Example 4.4.2 Determine the point of inflection and discuss the concavity of the following
functions.
a) f (x) = x4 + x3 – 3x2 + 1
Solution: - Find the second derivatives
f (x) = x4 + x3 – 3x2 + 1
f (x) = 12x2 + 6x – 6
f (x) = 6 (2x2 – x – 1)
f = 6 (2x – 1) (x + 1) = 0
Possible points of inflection occurs at x = -1 and x = -1/2 by testing the intervals at this points, we
can conclude that both are inflection points.
Interval - < x < -1 –1 < x < ½ ½<x<
93
Test value x = -2 x=0 x=1
Sign of f (x) f (-2) > 0 f (0) < 0 f (1) > 0
Conclusion concave concave concave
upward downward upward
Thus x = -1 and x = ½ are inflection points
Remark: - If f (x) = 0, even so the second derivative test does not apply, we use the first
derivative test.
Example 5.4.3
Let f (x) = x4 – 4x2 on [-3, 3]
a. Locate the local extrema, point of extrema and point of inflection.
b. Locate the interval where the graph of f is increasing, decreasing, concave upward and concave
downward.
c. Sketch the graph of f, determine the absolute maximum and minimum of the graph on [-3, 3].
Solution: -
a) f (x) = x4 – 4x2
i) f (x) = x4 – 4x2
x4 – 4x2 = 0
x2 – (x2 – 4) = 0
x2 (x + 2) (x – 2) = 0
x2 = 0 V x + 2 = 0 V x – 2 = 0
x = 0 V x = -2 V x = 2 are the zero of the function.
94
ii) f (x) = 4x3 – 8x
= 4x (x2 – 2) = 0
4x = 0 V x2 – 2 = 0
x= 2/3
f (- 2)>0
f ( 2)=0
f ( 2 ) > 0
95
Thus the function f is decreasing on the intervals (- , - 2 ) U (0, 2 ) and increasing on the
intervals (- 2 , 0) U ( 2 , ).
ii) Since the point x = 2 / 3 and x = - 2 / 3 are the zero of f (x), we have
f (- 2 ) = -8
f ( 2 ) = -8
The absolute maximum is 45 at x = 3 and x = -3 obtained at end points of the interval and the
In exercise 1 – 3 identify all relative extrema, use second derivative test when applicable.
1) f (x) = 6x – x2 3) f (x) = x2 + 3x - 8
4
2) f (x) = x +
x
In exercise 4 – 6 identify all relative extrema and point of inflection and sketch the graph.
4) f 9x) = x3 – 12x
96
1
5) f (x) = x2 +
x2
x
6) f (x) =
x 4
2
The most important application of derivative namely the theory of extrema, has a number of
application in economics and sundary business fields.
Now, we are going to present different examples that uses the previous concepts.
Example 5.5.1
a) Revenue
The market research department for a computer company used a large city to test market their new
product. They found that the relationship between price P (birr per unit) and the demand x (units
per week) was given by
P = 1,296 – 0.12x2 , 0 < x < 180
R (x) = px = 1,296x – 0.12x3, 0 < x < 180
i) Find the local extrema for the revenue function
ii) Over which intervals is the graph of the revenue function concave upward?
Concave downward?
Solution: -
Since the revenue function is given by
R (x) = 1,296x – 0.12x3
x2 = 3600
x = 60.
97
Thus x = -60 and x = 60 are the critical points. Since x = -60 (0,180), the only critical point
is at x = 60.
End points critical point end points
R (0) = 0 R (60) = 73, 440 R (180) = 194,400
Thus the relative minimum is at x = 0 and the relative maximum is at a point x = 180
b) To determine concavity we use the second derivative test
R (x) = 1296x – 0.12x3
R (x) = 1296 -= 0.36x2
R//(x) = -0.72x = 0
-0.72 x = 0
x=0
Interval -<x<0 0<x<
Test value x = -1 x=1
Sign of R (x) R (-1) > 0 R (1) < 0
Conclusion concave concave
upward downward
Thus the function R(x) is concave upward on the interval (- , 0) and downward on (0,)
b) Cost-Demand
A company manufacturers and sells x transistor radio per week. If the weekly cost and demand
equations are
C (x) = 5,000 + 2x
x
P = 10 - 0 x 8,000
1,000
For each week. Find
i) The maximum revenue
ii) The maximum profit
iii) The price that the company should charge for each radio
Solution: -
i) The revenue function received for selling x radios at $ p per radio is
R (x) = px
98
x
R (x) = (10 - )x
1,000
x2
R (x) = 10x -
1,000
So to find maximum revenue, we need to have
2x
R (x) = 10 -
1,000
x
R (x) = 10 -
500
x
10 - =0
500
x
- = -10
500
x
= 10
500
x = 5000
Hence x = 5000 is the critical point.
For absolute extrema use second derivative test.
x2
R (x) = 10x -
1,000
x
R (x) = 10 - < 0 for all x
500
Thus the maximum revenue is max R (x) = R (5000) = $ 25,000
ii) We know that
Profit = Revenue – cost
P (x) = R (x) – C (x)
x2
P (x) = (10x - ) – (5,000 + 2x)
1,000
x2
P (x) 10x - - 5,000 – 2x
1,000
x2
P (x) = 8x - - 5,000
1,000
99
To find the maximum profit use the first and second derivative test
2x
p (x) = 8 -
1,000
x
p (x) = 8 -
500
x
8- =0
500
x
- = -8
500
x = 4,000
thus x = 4,000 is the critical point
p (x) = 8 – x/500
p (x) = -1/500 < 0 for all x
Since x = 4,000 is the only critical value and p (x) < 0, the maximum profit is max p (x)
– p (4,000) = $ 11,000
iii) The maximum price is
x
p = 10 -
100
4000
p = 10 -
1000
p = 10 – 4
p=6
The maximum price is $6
c) Bacterial Control
A recreational swimming Lake is treated periodically to control harmful bacteria growth.
Suppose t days after a treatment, the concentration of bacteria per cubic centimeter is given by
C (t) = 30t2 – 240t + 500 for space 0 t 8
How many days after a treatment will the concentration be minimal? What is the minimum
concentration?
100
Solution: - We use first and second derivative test.
C (t) = 30t2 – 240t + 500
C (t) = 60t – 240
60t – 240 = 0
60t = 240
240
t= = 4 days
60
t = 4 days the critical point
The concentration will be minimal after 4 days. To find the minimum concentration.
C (t) = 30t2 – 240t + 500
C (t) = 60t – 240
C (t) = 60 > 0
Since t = 4 days and C (t) > 0
the minimal concentration is at t = 4
C (4) = 30 (4)2 – 240 (4) + 500
= 480 – 960 + 500
= 20 cm3
d) Average cost
If the average manufacturing cost (in dollars) per pairs of sun glasses is given y
C (x) = x2 – 6x + 12, 0 x 6
Where x is the number (in thousands) of pairs manufactured. How many pairs of glasses
should be manufactured to minimize the average cost per pair? What is the minimum
average cost per pair?
C (x) = x2 – 6x + 12
101
C (x) = 2x – 6
C (x) = 2 > 0
Since x = 3 is the critical value and C (x) > 0 thus 3 glasses must be produced to minimize the
One of the most familiar uses of a rate of change is to describe the motion of an object moving in
a straight line. The function f that gives the position of an object as a function of time t is called a
position function if over a period of time t, the object changes its position by the amount.
t = f (t + t) – f (t), change in distance
dis tan ce
rate =
time
The average rate of change in distance with respect to time is given by
change in dis tan ce f
=
change in time t
We call this average rate of change, “the average velocity (v)”
Definition: - If f (t) gives the position at time t of an object moving in a straight line, then the
average velocity of the object over the interval [t, t + t] is given by.
f f t t f (t )
V= =
t t
lim f t f (t 0 )
V (t0) =
t t0 t t0
Example 5.6.1
A small steel ball dropped from a tower will fall a distance of f(x) feet in x second given by
f (x) = 16x2, Find
i) The average velocity for the time 2 second, 3 second and 5 second.
f lim f t f (t 0 )
V= =
t t t0 t t0
102
V=
lim 16 t 2 t 02
t t0 t t0
lim 16t t 0 t t 0
V=
t t0 t t0
lim 16 t t 0
V=
t t0
V = 16 (t0 + t0)
V = 16 (2t0)
V = 32 t0
V (t) = 32t
For 2 second the average velocity is V (2) = 32 (2) = 64 f/s
For 3 second the average velocity is V(3) = 32 (3) = 96 f/s
For 5 second the average velocity is V (5) = 32 95) = 160 f/s
Definition 5.6.2: - If f is the position function for an object moving along a straight line, then the
acceleration of an object at a time t is given by
a (t) = V (t)
Where V (t) is the velocity of time t.
Example 5.6.2: - Find the acceleration of a Free-falling object whose position function is
f (t) = -16t2 + 100 feet
Solution: - the velocity of the function is given by
V (t) = f (t)
V (t) = -32t velocity function and the acceleration function is
a (t) = V (t)
a (t) = 32 feet / sec2
Remark: - Let f (t) is a position function
V (t) is a velocity function
a (t) is an acceleration function
V (t) = f (t)
a (t) = V (t)
a (t) = V (t) = f (x)
103
5.7 SUMMARY
1. [-1, 2]
104
2 28
f =0
6
2 28
f 0
6
2. Not differentiable at x = 0
3. f (x) = 0 only at x = 0
4. [1, 2]
f (3/2) = 0
5. [-1, 3]
f (-1) = 0
f (5/3) = 0
6. f (-1/2) = -1
2 6 2
7. f = /3
6
8. f (-/2) = f (/2) = 1
9. f (8/27) = 1
10. f ( 3 ) = f (- 3 ) = -3
105
2. Relative minimum is at x = -2 and f (-2) = -4
Relative maximum is at x = 2 and f (2) = 4
3. Relative minimum is at x = -3/2 and f (-3/2) = -41/4
4. Relative maximum is at x = -2 and f (-2) = 16
Relative minimum is at x = 2 and f (2) = -16
106
5.9 MODEL EXAMINATION QUESTIONS
In exercise 1 – 4 find the interval where the following functions are increasing, decreasing and
local extrema.
1) f (x) = x2 – 16x + 12 3) f (x) = x2 + 6x + 7
2) f (x) = 4 + 10x – x2 4) f (x) = 5 + 8x – 2x2
In exercise 5 – 7 find all local maxima and minima using the second derivative test when ever it
applies. If it fails, use first derivative test.
5) f (x) = 2x2 – 8x + 6
16
6) f (x) = x +
x
7) f (x) = 6x – x2 + 4
8) The concentration c (t) in milligrams per cubic centimeter of a particular drug in a patients
blood-stream is given by
0.16t
c (t) =
t 4t 4
2
Where t is the number of hours after the drug is taken. How many hours after the drug is given
will the concentration be maximum? What is the maximum concentration?
9) A company manufactures and sells x television set per month. The monthly cost and demand
equations are
107
Unit 6: Integration and its application
CONTENTS
6.0 Aims and Objectives
6.1 Introduction
6.2 Antiderivatives
6.3 Intermediate Value Theorem for Continuous Function
6.4 Partition of [a, b], their fitment area of Inscribed and Circumscribed Rectangular Polygon
6.5 Lower and Upper Sum
6.6 Definition of Riemann Integral
6.7 Summary
6.8 Model Examination Questions
6.9 Answers to Check Your Progress
6.1 INTRODUCTION
In geometric terms finding derivatives in equivalent to finding the slope of a line tangent to a curve.
The main part of this unit is focused on explaining the integral calculus, viewed geometrically, is
concerned with computing areas. You will see how area is estimated by using Remann sum and
also fundamental theorem of calculus, properties of integral will also be discussed.
6.2 ANTIDERIVATIVES
108
Definition 6.2.1
A function F(x) is an antiderivative of a function f(x) if F/(x) = f(x), for all x in the domain of f.
Notation: The set of all antiderivative of f is the indefinite integral of f with respect to x, is denoted
by f(x) dx
The symbol in an integral sign, the function f is the integrand of integral and x is the variable of
integration.
N o te
- Suppose the function F(x) is an antiderivative for the function f(x) on the interval I. i.e.,
assume that F/(x) = f(x) for all x in I then any antiderivative G(x) for f(x) on I must have the
form G(x) = F(x) + c for some constant c, and every function of this form is an antiderivative
for f(x).
- The only antiderivative of the zero function are the constant function.
i.e., if F/(x) = 0 x in its domain, then (x) = c, c R
Example 6.2.1
Evaluate 2x dx
antiderivative of 2x
Solution
2x dx = x2 + c
109
x n 1 x n 1 n
1. xn dx = + c, n -1, n Q d
= x
n 1 dx
n 1
cos kx
2. Sin kx dx = c Cos kx
k
d
dx = sin kx
k
sin kx
3. Cos kx dx = c sin kx
k d
dx = cos kx
k
4. sec2x dx = tan x + c d
dx
(tan x) = sec2x
5. csc2x dx = - cot x + c
d
dx (- cot x) = csc2x
Solution: Solution:
x6 1
x5dx =
6
c x
dx = x - ½ dx
= 2x½ + c
=2 x+c
c) sin 2x dx d) cos x
2 dx
Solution: Solution:
Cos 2 x
sin 2x dx = c Cos x
2 dx = cos 1
2 xdx
2
1
sin 2 x
= 1
+c
2
= 2 si n ½ x + c
x
= 2 si n 2 +c
Note. Evaluating an integral formula can sometimes be difficult, but checking it, once found, is
relatively easy.
Example 6.2.3. Show that
110
x cosx dx = x sin x + cos + c
Solution: d
dx
(x sin x + cos x + c) = x cos x + sin x – sin x + 0
= x cos x
: . x cos x dx = x sin x + cos x + c
Rules for indefinite integration (Antiderivative)
1. Constant Multiple Rule: kf(x)dx = k f(x) dx
2. Rule for Negative: -f(x)dx = - f(x) dx
3. Sum and difference Rule:
[f(x) g(x)] dx = f(x) dx g(x) dx
Example 6.2.4
Find [12x2 – 2x3] dx
Solution. 12x2 – 2x3 dx = 12x2dx = 2x3dx
= 12 x2dx - 2 x3dx
x3 x4
= 12 - 2 + c
3 4
1 4
= 4x3- x +c
2
Example 6.2.5
Evaluate x2 + 3x + 4 dx
Solution
x2 + 3x + 4 dx = x2 dx + 3x dx + 4 dx
x3 x2
= 3 4x c
3 2
1 3 3 2
= x + x + 4x + c
3 2
111
6.3 INTERMEDIATE VALUE THEOREM FOR CONTINUOUS FUNCTION
Recall that a function is called continuous if it is continuous for every number in its domain. A
function that is not continuous through its domain may still be continuous if it is restricted to some
particular intervals that are subsets of the domain. If a function is continuous in the given interval
then we can say that it is defined for all number in the interval.
The function f(y), being continuous on [a, b], take any value between f(a) and f(b)
Example 6.3.1
Use the intermediate value theorem to determine whether the given function have a zero in the
given interval.
f(x) = x3 + 3x2 – x – 3, [-2, 0]
Solution
We need to show there exist c [-2, 0] with f(c) = 0 first evaluate f(-2) & f(0) and compare the
value
f(-2) = (-2)3 + 3(-2)n – (-2) – 3
=3>0
112
f(0) = (0)3 + 3(0)2 – (0) – 3
= -3 < 0
Since f(0) < 0 < f(-2), by the intermediate value theorem there is c [-2, 0] with f(c) = 0
Note. In doing problem like the one in the example 6.3.1, we follow the steps
1. Check function value of end point 5.
2. If they are of opposite sign, then, we must have f(c) = 0 somewhere in the interval.
6.4 Partition of [A, B], their fitment area of inscribed and circumscribed rectangular polygon
Area problem: Let f(x) be a continuous function for all x in [a, b]. Find the area of the region R
bounded by the graph of f(x), the x-axis and the line x = a and x = b
To deal with the area of R we use the idea of approximating the region R by fitting R with inscribed
or circumscribed rectangles and we will be able to calculate the limit of the sequence of area of
these rectangle for approximation.
Given an arbitrary continuous function y = f(x) on an interval [a, b], we partition the interval into
n-subintervals by choosing n – 1 points, say x1, x2, ---, xn-1 between a and b subject only to the
condition that
113
a < x1 < x2 < ---, xn-1 < b
we usually denote a by xo and b by xn
The set p = {x0, x1, x2, ---, xn} is called a partition of [a, b]
X0 = a x1 x2 x3 xk-1 xk xn = b
114
ba
Since we use n-rectangles of equal width, the width of each is x = and the end points of
n
the resulting subintervals are xo = a, x1 = a + x, x2 = a + 2 x ---, xn = a +
(n-1) x = b. The height of the rectangle constructed over the interval [xk-1, xk] to be the minimum
value of f(x) on [xk-1, xk]. Since f(x) is continuous on [xk-1, xk] there is at least one number Ck
[xk-1, xk] with
f(Ck) = Minimum {f(x)| xk-1 x xk}
= A1 + A2 + --- + An
= f(c1) x + f(c2) x + --- + f(cn) x
n
Sn =
k 1
f(Ck) x
Note. The lower approximating sum, each of the associated rectangle lie entirely with in the region
R our definition of area should provide that
Sn A
we obtain what is called an upper approximating sum, denoted by S n . i.e. we partition [a, b] into
ba
n-subintervals of equal length x = , with end points
n
x0 = a, x1= a + x, ---, xn = a + (n – 1) x, we take the height of the rectangle over the interval
[xk-1, xk] to be the value f(dk), where f(dk) = Maximum {f(x)| xk-1 x xk}
116
n n
Sn= Ak f (d k )x
k 1 k 1
Example 6.5.2. Find the upper approximating sum S 4 for the area of the region R, which is given
in the example 6.5.1 above
Solution:
b a 1 0
n = 4, x = 1
4
n 4
there are n – 1 = 3 intermediate value i.e.,
a = x0 = 0, x1 = ¼ x2 = ½ x3 = ¾ x4 = 1 = b
A1 = f(x1) x = f (¼) ¼ = 17
64
A2 = f(x2) x = f (½) ¼ = 5
16
A3 = f(x3) x = f (¾) ¼ = 25
64
A4 = f(x4) x = f(1) ¼ = ½
4
94
S 4 Ai
i 1 64
Since, for upper approximating sums, the union of the circumscribed rectangle entirely contains
Note. A
Sn Sn
The area A of R is the unique number satisfying Sn A S n for all lower and upper approximating
sums.
Example 6.5.3
Let R be the region bounded above by the graph of f(x) = x2, below by the x-axis, on the left by x
= 0, and on the right by x = 1. Show that the number 1
3 satisfies the inequality Sn 1
3 S n for
all lower and upper approximating sums we how
117
lim lim
Sn = 1
3 = Sn
n n
Solution:
We shall make use of the formula
k (k 1)(2k 1)
1 + 22 + 32 + --- + k2 =
6
To form the lower approximating sum for f(x) = x2 on [0, 1], we use n-subintervals of equal size
x = 1
n , and end points x0 = 0, x1 = 1
n , x2 = 2
n , x3 = 3
n - - -, xn = n
n =1
Since f(x) = x2 is increasing on [0, 1], the minimum value of f(x) on each subinterval
k 1
[xk-1, xk] will occur at the left end points, xk-1 i.e., Ck = xk-1 = for k = 1, 2, --- n
n
1 1 1 n 1 1
Sn = f(0) + f( ) + - - - + f
n n n n n
1 2 n 1 2 1
= [02 + ( )2 + ( )2 + - - - ]
n n n n
1
= [02 + 12 + 22 + - - - (n-1)2]
n3
(n 1)((n 1) 1)(2(n 1) 1 1
= n 3
6
(n 1)n(2n 1)
=
6n 3
2n 2 3n 1 1 3n 1
Sn = - - - (*)
6n 3 3 6n 2
To obtain the upper approximating sum we use dk = xk on the interval [xk-1, xk],
k = 1, 2, ---, n i.e., dk in the right end point of the kth interval. Thus,
n 1 1
S n f 1n f 2 n f f n n
1 1 1
n n n n n
n 1 1 n 2 1
2
= n
1 2
. 2 n
1 2 1
n
n n n n n
118
1
= [12 + 22 + 32 + - - - + (n – 1)2 + n2]
n3
n(n 1)(2n 1)
=
6n 3
1 3n 1
: - Sn - - - (* *)
3 6n 2
1 3n 1
The upper approximating sum S n for f(x) x2 on [0, 1] is S n =
3 6n 2
1 3n 1 1 1 3n 1
From equation (*)&(* *) we have 2
= Sn S n = for all n 2
3 6n 3 3 6n 2
lim Sn lim 1 3n 1 1
= = and
n n 3 6n 2 3
lim S n lim 1 3n 1 1
= 3 6n 2 = 3
n n
1
: - We define the area of the region R to be the number A=
3
Theorem 6.5.1
Let f(x) be continuous on the interval [a, b]. Let Sn and S n denote the lower and upper
lim Sn lim S n
approximating sum for on [a, b]. Then and exists and
n n
lim Sn lim S n
=
n n
2. Find the upper approximating sum S 4 for the region R in Question 1 above
3. Let f(x) = 3x + 2
a) Find the lower approximating sum S4 for f(x) on [0, 2] with n = 4 subinterval
b) Find an expression for the lower approximation sum Sn for f(x) on [0, 2] with n-
subintervals.
119
lim Sn
c) Find with Sn as in part (b)
n
d) Find an expression for the upper approximation sum S n for f(x) on [0, 2] with n-
subintervals.
Definition 6.6.1. The norm of the partition Pn, denoted by ||Pn||, to be the largest of the length of
the subintervals. i.e. ||Pn|| = Max {(x1 – x0), (x2 – y1), - - - (xn – yn-1)} then xk – xk-1 ||Pn||, k = 1,
2, - - - , n
Definition 6.6.2.
Let f(x) be continuous on [a, b]. a Remann sum for f(x) on [a, b] is any sum of the form
n
Rn = f (t
k 1
k )x k , tk [xk-1, xk]
where {x0, x1, x2, …, xn} is a partition of [a, b], xk = xk-1 and tk [xk-1, xk], k = 1, 2, 3, …, n
Theorem 6.6.1
Let f(x) be continuous on [a, b]. There exists a unique number I with
n
lim Rn lim
I=
n
=
n
f (t
k 1
k )x k , tk [xk-1, xk]
For all Riemann sums Rn with partition Pn = {x0, x1, - - -, xn} on [a, b] where
xk = xk – xk-1 and ||Pn|| 0 an n .
We call the number I in theorem 6.5.1 the definite integral of f over [a, b], we say that f is integrable
over [a, b], and we say that the Riemann sums of f on [a, b] converge to the number I.
120
b
We write I an f ( x)dx , which is read “integral of f from a to b” if the limit exists i.e.,
a
b
lim n
p 0 f ( x)dx
f (t k )x k =
k 1 a
Definition 6.6.3
If f is continuous on [a, b] and Lf = Uf = I then we say that
i) f is integrable on [a, b]
ii) the definite integral of f(x) from x = a to x = b is I
iii) I is expressed in symbols by the equation
b
I= f ( x)dx
a
iv) If f(x) 0 for each x in [a, b], then the area, A bounded by the curve y = f(x), y=0, x = a
b
and x = b, is defined to be the definite integral of f(x) from x = a to x = b, i.e., A = f ( x)dx
a
b
Notation. Terminology associated with the symbol f ( x)dx
a
Integrand
Upper limit of integration
x is the variable of integration
b
Solution:
We evaluate the integral for the area as a limit
of Riemann sums. Partition [0, b] into
b0 b
n-subintervals of length x = .
n n
The point of the partition are:
x0 = 0, x1 = x, x2 = 2x, - - -, xn-1 = (n – 1) x, xn = nx = b
Let us choose each Ck to be lub of each subinterval (we are free to choose Ck as we like)
Thus c1 = x1, c2 = x2, c3 = x3 - - - cn = xn
A1 = -f(c1) x = f(x) x = (x)2 x = 12 (x)3
A2 = f(c2) x = f(2x) x = (2x)2 x = 22 (x)3
A3 = f(c3) x = f(3x) x = (3x)2 x = 32(x)3
An = -f(cn) x = f(nx) x = (nx)2 x = n2 (x)3
n
= k
k 1
2
(x) 3
= bn 3 . n(n 1)(2n 1) , x = b
n
6
b 3 (n 1)(2n 1)
= .
6 n2
122
=
b3
6
. 2 n3 n12
b
lim n
p 0
Use the definition of definite integral f ( x)dx = f (c k )x to find the area under the
a k 1
parabola from x = 0 to x = b
b
lim S n
x
2
dx =
a n
=
lim b3
. 2 n3 n12
n 6
b3
= ( 2 + 0 + 0)
6
b3
=
6
Function with no Riemann integral
Some discontinuous function are integrable, others are not
Example 6.7.2
f(x) = 1 when x is rational
0 when x is irrational
f(x) has no Riemann integral over [0, 1] for any partition p of [0, 1], the upper and lower sum are
Uf = Mk - xk = 1 - xk = xk = 1
Lf = mkxk = 0.xk = 0
For the integral of f to exist over [0, 1], U and L would have to have the same limit as ||p||0. But
lim L 0 lim U 1
they do not: while
p 0 p 0
123
CHECK YOUR PROGRESS 6.6.1
Use a definite integral to find the area of the region between the given curve and the x-axis on the
interval (0, b]
a) y = 3x2 b) y = 2x
a b
2. Order of integration:
b
f ( x)dx = - f ( x)dx
a
b b
3. Constant multiples: k f ( x)dx = k f ( x)dx , k R
a a
b b b
4. Sum and differences: [ f ( x) g ( x)]dx = f ( x)dx g ( x)dx
a a a
b c c
5. Additivety:
a
f ( x)dx +
b
f ( x)dx = f ( x)dx
a
b b
7. Domination: f(x) g(x) on [a, b] f ( x)dx g ( x)dx
a a
b
f(x) 0 on [a, b] f ( x)dx
a
0
Example 6.6.3
1 4 1
Suppose f ( x)dx = 5
1
f ( x)dx = -2
1
h( x)dx = 7 then
1
124
1 4
1.
4
f ( x)dx = - f ( x)dx = - (-2) = 2
1
1 1 1
2. 2 f ( x) + 3h(x) dx = 2 f ( x)dx + 3 h( x)dx
1 1 1
= 2(5) + 3(7)
= 10 + 21
= 31
4 1 4
3.
1
f ( x)dx =
1
f ( x)dx + f ( x)dx
1
= 5 + -2
=3
f ( x)dx = -4
1
f ( x)dx
1
=6 g ( x)dx
1
=8
5 5
e) [ f ( x) g ( x)]dx f) [4 f ( x) g ( x)]dx
1 1
f ( x)dx = f(c) (b – a)
a
125
b
m(b – a) f ( x)dx M (b – a)
a
b
1
and m
ba f ( x)dx M
a
By the intermediate value theorem for continuous functions, there exists some c such that
b
1
f(c) =
ba f ( x)dx
a
b
and f ( x)dx = f(c) (b – a)
a
for each x in [a, b], then g(x) is continuous on [a, b] and differentiable on (a, b) and for all x in (a,
b), g/(x) = f(x) i.e.,
d
x
f (t )dt = f(x)
dx a
1
x h x
lim
= f (t ) dt f (t )dt
h0 h a a
1
x x h x
lim
h 0 h a
= f (t )dt f (t )dt f (t )dt
a a
126
1
x h
lim
= f (t )dt
h0 h a
lim 1
= [f(c) (x + h – x)] by mean value theorem
h0 h
lim f (c )
for some c between x and x + h
h0
Since f is continuous on [a, b] and c is between x & x + h it follows that
lim f (c )
g/(x) = = f(x) for all x such that a < x < b This concludes the proof.
h0
x
d
dx a
Note. f (t )dt = f(x), has a nice geometric interpretation the integral of f from a to x is the area
A(x) of the region between the graph of f and the x-axis from a to x.
Proof. Let f and g be continuous on the closed and bounded interval [a, b] and for each x in [a, b],
x
let G(x) = f (t )dt
a
Then by the theorem above G/(x) = f(x) on [a, b], since G/(x) = g(x) for all x on [a, b], there exist
some constant c such that
G(x) = g(x) + c, for all x on [a, b]
Since G(a) = 0, we get c = -g(a). Then
b
f ( x)dx = G(b)
a
= g(b) + c
127
= g(b) – g(a)
This complete the proof
b
Note. How to evaluate f ( x)dx
a
1. Find an antiderivative F of f
2. Calculate the number F(b) – F(a).
b
This number will be f ( x)dx
a
Example 6.6.4
Compute each of the following definite integral
4
a) x 2 dx
0
4
x3 43 03
Solution: x 2 dx 4
0
0
3 3 3
64
=
3
2
b) Cos x dx
2
2
Cos x dx = Sin x
Solution: 2
2
2
= S in
2 - S in ( 2 )
= 1 – (-1)
=2
3 4
4
c) x 2 dx
1 x
2
Solution: x 4 4 x
3 4
1
4 3
2
4 4 - (1 + 4)
9–5
4
128
0
d) ( x 3 x 2 2 x)dx
1
x 4 x3 2x 2
Solution: 0
1
4 3 2
0- 14 13 1
5
12
Note. In order to use the fundamental theorem, it is necessary to be able to find an antiderivative
F(x) for f(x). Here is a summary list of the antiderivative formulas.
m dx = mx + c, mR Sec2x dx = tan x + c
x n 1
xn dx = + c, n -1 csc2x dx = -co + x + c
n 1
sin dx = -cos x + c sec x tan x dx = sec x + c
cos x dx = sin x + c csc x cot x dx = -csc x +c
a
f ( g ( x)) . g/(x) dx = f (u)du
g (a)
Example 6.6.5
3
Evaluate
0
Cos x sin x dx
Solution.
Let U = cos x then du = -sin x dx
When x = 0, then u = cos 0
129
When x = , then u = cos
3 3
cos 3 1
2 32
3 2 1
u du u du
2
Cos x sin x dx = u
0 cos 0 1
3 1
2 3 1
2
3
2
1 2
3
2 3 1
2
3
2
1
Example 6.6.6
1
Evaluate 3x 2 x 3 1 dx
1
Solution. Let U = x3 + 1
du = 3x2
1 u (1)
3x x 1 du
2 3
u du
1 u ( 1)
u (1)
2 32
= u c
3 u ( 1)
2 3 1
= ( x 1)
3 1
2 3 32 2
(1 11) ((1) 3 1) 2
3
=
3 3
2 3
2
= (2)
3
2
= (2 2 )
3
4 2
=
3
2
Example 6.6.7. Evaluate sin (3x)dx
0
130
2 3( 0 )
sin (3x)dx =
0
3( 0 )
1
3 sin u du
1
cos x
6
=
3 0
1
= (-cos (6) – (- cos (0)
3
1
= (1 – cos 6)
3
2 0 1
d)
0
4 x 1 dx e) x(
2
x 1 dx f) 2 x( x 2 1) 2 dx
0
2
1 2 3
cos x
g) ( x 2 x 3)dx
2
h) 2 dx i) sin(2 x)dx
0 sin x 0
4
2 1 1
dx
j) x 4 x 2 dx x 1 2x 1
2
k) dx l) 3
1 1 0
6.7 SUMMARY
131
The function F(x) is called an antiderivative for the function f(x) if F/(x) = f(x). The most general
antiderivative for f(x), if one exist, is F(x) + c where F/(x) = f(x) and c is an arbitrary constant. I.e.
f(x)dx = F(x) + c
- If f(x) is continuous on [a, b], the definite integral of f(x) on [a, b] is defined to be the limit
of the approximating Riemann sums,
b n
lim
i.e., f ( x)dx =
n
f (t
k 1
k )x k , tk [xk-1, xk]
a
b
- If f(x) 0 on [a, b] and f(x) is continuous, then f ( x)dx is the area of the region bounded
a
x
x 2 dx b) Sin 4x dx
2 5
a) 3
c) (x2 – 6x)2 dx d) x 2 1 dx
2. Let f(x) = 3x + 4 and the interval [1, 2] consider the region R bounded by the graph of f(x),
the x-axis and between [1, 2]
Find a) S6
b) S 6
132
c) an expression for Sn and S n
3. Use a definite integral to find the area of the region between the given curve to the x-axis on
the interval [0, n]
x
a) f(x) = 5x2 b) f(x) = +1
3
3 3
4. Given that f ( x)dx = 5 and g ( x)dx = -2, find
1 1
3 3
a) [ f ( x) g ( x)]dx b) [6 g ( x) 2 f ( x)]dx
1 1
1 3 3
c) 2 f ( x)dx g ( x)dx d) (2 f ( x) 3)dx
3 1 1
x x
4 1 5
x 2 dx 2 dx
2 1 2
a) x dx b) 3
c)
0 0 3
4 2 5
d) sin x dx e) ( x 4) dx 10
f) 3x 2 2 dx
0 1 1
133
b
Area = 3 x 2 dx = b3
a
b
b) Using n-subintervals of length x = and right-end point value
n
b
Area = 2 x dx = b2
a
10 8 2 7 7
e) f) g) h) 2 1
5 3 3
3 8 2
i) j) 3 k) l)
4 3 9
134
Unit 7: Application of Integrals
CONTENTS
7.0 Aims and Objectives
7.1 Introduction
7.2 Areas, Volumes, Work and Arc Length
7.2.1 Area between Curves
7.2.2 Volumes
7.2.3 Work
7.3.4 Arc Length
7.3 Summary
7.4 Answers to Check Your Progress
7.5 Model Examination Questions
This unit aims at highlighting the importance of integration in finding areas, volumes and work
done at various application problems. It also describe different techniques of integration in solving
problems. After this unit, you should be able to:
find the area at the region bounded by graph
use the concept of integration to calculate areas of triangles
calculate consumer surplus and producer surplus given supply and demand curves
calculate the volume of solid region formed by revolving different regions
find the work done required to do different jobs
calculate the are length for different graphs given their intervals.
7.1 INTRODUCTION
When we introduced integration in the previous section. This book introduced that area is only one
of the many applications of the definite integral. But there are different applications of intervals.
Among these application volume, work done and arc length are few of them. The scope of this
unit is to enable students to use the idea of integration in solving different problems. So this unit
tries to define areas, volumes, work done and arc length in combination with application problems.
135
There are different solved examples and problems that a student should follow smoothly to capture
the concept of integration in different application problems.
Example 7.2.1.1: To find the area between the functions f and g we proceed as follows
To find the area of the region between f and g bounded by x = a and x = b we use
b
f ( x) g ( x)dx
a
Therefore the area between two curves f and g from the above examples
b b b
f ( x) g ( x)dx = f ( x) dx - g ( x) dx
a a a
136
If f and g are continuous on [a, b] and g(x) f(x) for all x in [a, b]. The area of the region bounded
by the graphs at f and g and the vertical lines x = a and x = b is
b
A: f ( x) g ( x)dx
a
Example 7.2.1.2. Find the area of the region bounded by the graph of
a) f(x) 2 – x2 and g(x) = x
Solution: - The graph of f and g have two points of intersection. To find these points of intersection
on the x-coordinate we have to equate the function.
f(x) = g(x)
2 – x2 = x
2 – x2 – x = 0
-x2 – x + 2 = 0
(-x + 1) (x + 2) = 0
-x + 1 = 0 v x + 2 = 0
x = 1 v x = -2
We have a = -2 and b = 1, since g(x) f(x) on the interval [-2, 1]
So the area of the shaded region is
b
A= f ( x) g ( x)dx
a
b
A = (2 x 2 x)dx
a
1
(2 x x)dx
2
A=
2
1 1 1
2dx x dx x dx 0
2
A=
2 2 2
3 2 1
1 x 1 x
A = 2x
2 3 2 2 2
A = (2 + 4) – 13 8 3 1 2 4 2
A=6–3+ 3
2
A= 9
2
137
Solution: To find the points of intersection of f and g we have
f(x) = g(x)
sin x = cos x
sin x cos x
cos x cos x
sin x
1
cos x
tan x = 1
x = tan-1 1
x=
4 , 5
4 on the interval 0 x 2 so a =
4 and b = 5
4 sin x cos x on the interval [
, 5 ], the area of the region is
4 4
5
4
5 5
4 4
A= sin x dx cos x dx
4 4
5
4 4
A = cos sin x
4 4
A = - 2 2 2 2
2 2 2 2
A= 2 + 2 + 2 + 2
2 2 2 2
A= 4 2 = 2 2
2
Solution: f and g don’t have intersection points here so let us sketch the graph on the interval [-2,
1].
138
Where as a = -2 and b = 1
We observe that f(x) g(x) on the interval [-2, 1] such that a = -2 and b = 1. Thus
b
A= f ( x) g ( x)dx
a
1
A= 1
2 x 3 ( x 2 1) dx
2
1
A= 1
2 x 3 x 2 1 dx
2
1
A= 1
2 x x 2 2 dx
2
1 1 1
xd x dx 2dx
2
A= 1
2
2 2 2
x2 1 x3 1 1
A= 2x
4 2 3 2 2
A= 14 4 4 13 8 3 + (2 + 4)
A= 3
4 93 + 6
A= 3 +3+6
4
A= 3 +9
4
A = 33
4
d) Using data from the first 3 years production as well as geological studies, the management
of an oil company estimates that oil will be pumped from a producing field at a rate given by
100
R(t) = 10 , 0 t 15
t 10
139
Where R(t) is the rate of production in thousands of barrels per year, t years after pumping begins.
Find the area between the graph R and the t-axis, over the interval [5, 10] and interpret.
Solution: - We can observe that t-axis is similar to x-axis and R(t) is similar to f(x) where a = 5
and b = 10, thus the area of the region is
b
A = R(t )dt
a
100
10
A = 10 dt
5
t 10
10 10
100
A= dt 10dt
5
t 10 5
Now let x = t + 10
dx = dt
100
10 10
A = dt 10 dt
5 t 10 5
10
100
A=
5
x
dx 10t 10
5
10
1
A = 100 dx 10(10 5)
5
x
10
A = 100 lnx + 50
5
10
A = 100 ln (t + 10) + 50
5
140
Given that demand function D(x) = 50 – ½ x
Supply function S(x) = 1
8 x
Solution: First we have to find the equilibrium point xo and po, At equilibrium point
D(x) = S(x)
50 – ½ x = 1
8 x
50 = 1
8 x+½x
50 = 5
8 x
5
8 x = 50
x = 50x 5
8
x = 80
and D(80) = 10 thus po = 10 and xo = 80
S(80) = 10
The consumer surplus region is
A = Area under demand curve – the rectangular region po and xo
80
A= D( x)dx p x
0
0 0
80
A= 50
0
1
2 x dx – (10) (80)
80 80
A = 50dx 1 2 xdx 800
0 0
80 x2 80
50x 800
0 4 0
141
Similarly the producer surplus is
A = Area of rectangular region poxo – Area under supply curve
80
A = poxo - S ( x)dx
0
80
A = poxo -
0
1
8 x dx
x2 80
A = 800 -
16 0
(80) 2
A = 800 -
16
A = 800 – 400
A = 400
The producer surplus is 400
Rectangle
R
Axis of revolution
142
The volume of such a disc is given by
V = R2w
where V is the volume of the disc
R is the radius of the disc
w is the width
a a
Example 7.2.2.1. Find the volume of the solid formed by revolving the region bounded by f(x) =
2 – x2 and g(x) = 1 about the line y = 1
Solution: Find the intersection point of f and g
i.e., f(x) = g(x)
2 – x2 = 1
-x2 = 1-2
-x2 = -1
x2 = 1
x=1
143
To find the radius we subtract g(x) and f(x)
R(x) = f(x) – g(x)
= (2 – x2) – 1
= 2 – x2 – 1
= -x2 + 2 – 1
= -x2 + 1
Finally, use integration between –1 and 1
b
V = R( x) dx
2
V = x 2 1 dx
1
2
1
V = x 4 2 x 2 1dx
1
1
x5 2x 3 1
V = x
5 3 1
16
V =
15
16
V= 3.35
15
R
144
r
Axis of revolution
w
r
Axis of revolution
b
V = R 2 ( x) r 2 ( x) dx
a
b b
V = R 2 ( x)dx r 2 ( x)dx
a a
Example 7.2.2.2. Find the volume of the solid formed by revolving the region bounded by the
graph of y = x and y = x2 about the x-axis
145
We have R(x) = x outer radius
r(x) = x2 inner radius
integrating between 0 and 1, we have
b
V = R 2 ( x) r 2 ( x) dx
a
x x dx
1
V =
2 2 2
0
1
V = x x 4 dx
0
x2 x5 1
V =
2 5 0
V = 1 2 15
V = 3
10
Definition 7.2.2.3. (Solid with known cross section)
With the disc method, It is possible to find the volume of a solid having circular cross section.
Whose area is A = R2 and generalize the method to solids of any shape, here we should know a
formula for the area of any cross-section like squares, rectangles etc.
146
Example 7.2.2.3. Find the volume of the solid whose base is the area bounded by the lines
f(x) = 1 - x
2 g(x) = -1 + x
2 and x = 0 and whose cross sections perpendicular to
the x-axis are equilateral triangles.
3
Area = (base)2, for area of equilateral triangle
4
3
Area = (2-x)2, area of cross section
4
2
3
V=
0
4
(2 – x)2 dx
147
2
3
V=
4
0
(2 – x)2 dx
3 (2 x) 3 2
V= 0
4 3
3 8
V= 3
4
2 3
V=
3
7.2.3 Work
The concept work is important for determining the energy needed to perform various physical
tasks. For instance, you may need to know the work done when a spring is compressed, a rocket
is launched etc. We may say a work is done when a force moves an object.
Definition 7.2.3.1. If an object is morning along a straight line by a continuously varying force
F(x) the work done w by the force as the object is moving from x = a to x = b is given by
d
W = F ( x)dx
a
Remark:
1. The force required to compress or stretch a spring (with in its elastic limits) is proportional
to the distance d that the spring is stretched or compressed.
i.e., F = kd, where k is the specific nature of the spring.
2. The force of attraction between two particles of masses m1 and m2 is given by
m1 m2
F=k
d2
Where k is a constant k = 6.670 x 10-8 and d is the distance between the two particles.
148
Solution: Using remark 1, F(x) is the force required to compress x units (from its natural length)
F(x) = kx, we have
F(3) = k(3)
750 = 3k
750
k=
3
k = 250 and
F(x) = kx
F(x) = 250x, to find the increment of the work done. Since the spring is compressed from
x = 3 to x = 6
b
W = F ( x)dx
a
6
W = 250 x dx
3
250x 2 6
W=
2 3
250(6) 2 250 2
W= 3
2 2
W = 4500 – 1125
W = 3375 in lb
149
If the function given by y = f(x) has a continuous derivative f1 on the interval [a, b] then the arc-
length of f between x = a and x = b is given by
d
S=
c
1 [ f ( x)]2 dx
Similarly, for a smooth curve given by x = g(y) the arc-length of g between c and d is
b
S=
a
1 g ( y ) dy
150
sin x
2
[f/(x)]2 =
cos x
sin 2 x
=
cos 2 x
= tan2x
the arc length
b
S=
a
1 [ f ( x)]2 dx
4
S=
0
1 tan 2 x dx , since 1 + tan2x = sec2x
4
S=
0
sec 2 x dx
4
S = sec x dx
0
4
S = ln sec x tan x
0
S = ln
2 1 - ln 1
S = 0.8814
The arc length is 0.8814
b) f(x) = x between x = 0 and x = 1
Solution:
Since f(x) = x
f/(x) = 1
[f/(x)]2 = 12 = 1
Now the are length is
b
S=
a
1 [ f ( x)]2 dx
151
1
S=
0
1 1 dx
1
S=
0
2 dx
1
S= 2
0
S= 2 (1 – 0)
S= 2
1
[f/(x)]2 = 2 x 3
3
49 x
2 2
3
4
2
9x 3
the arc length is
b
S=
a
1 [ f ( x)]2 dx
8
4
S = 1 2
dx
1 9x 3
9x 3 4
8 2
S=
1 9x 3
2
dx
9x 3 4
8 2
S=
1 3x 3
2
dx
Now let U = 9 x 3 4
2
du = 9 2 3 x dx
1
3
1
du = 6 x 3
dx
6dx
du = 1
x3
152
du dx
1
6 x3
Now using substitution method, we have
4 9x
8 2
3
S=
1
1
3x 3
dx
40
u du
S=
13
3 6
40
1
S= u du
18 13
1
S = 2 U 3
18 3
2
40
13
1
2
40
S= U 3
27 13
S=
1
27
40 3 13 3
2 2
153
9. y = 2x2, x = 1, y=0
10. y = 4 – x x = 0, y=0
11. y = cos x x= , y = 0, x = 0
2
3
13. f(x) = cos h, on the interval [0, 2]
14. f(x) = 3x + 4, on the interval [1, 5]
7.3 SUMMARY
1. The area of a given region between curves f and g such that f(x) g(x) for all x in [a, b] is
given by
b b b
A= f ( x)dx g ( x)dx f ( x) g ( x)dx
a a a
d
Volume V = R( y ) dy for vertical axis
2
b b
V = R( x) dx [r ( x)]2 dx
2
a a
4. The volume of solid with known cross section such that the area A(x) taken perpendicular
to
a) the x-axis is
b
V= A( x)dx
a
154
b) the y-axis is
b
V= A( y)dy
a
5. The work done W by the variable force F(x) as the object is moved from x = a to x = b is
b
W= f ( x)dx
a
6. If f(x) has a continuous derivative f/(x) on the interval [a, b], then the arc length S of f(x)
between x = a and x = b is
b
S=
a
1 [ f ( x)]2 dx
4. 15 5. 9 6. 4
3
7. a) Consumer surplus = 50, 000
b) Producer surplus = 25, 479
8. 8 9. 4 10. 64
3 5 3
11.
2
4
12. 2 5 5 1
3
13. sinh 2 kjj,, 14. 4 10
In exercise 1.-5 sketch the region bounded by the graphs of the given functions and find the area
of each region.
1. f(x) = x2 – 4x, g(x) = 0
2. f(x) = x2 + 2x + 1, g(x) = 3x + 3
3. y = x, y = 2 – x, y=0
4. f(x) = 3x2 + 2x, g(x) = 8
5. f(x) = x2 – 4x + 3, g(x) = 3 + 4x – x2
155
In exercise 6-9 find the volume of the solid generated by revolving the region bounded by the
graph of the given equation about x = 6
6. y = x, y = 0, y = 4, x = 6
7. x = 6, y = 0, y = 4, x = 0
8. x = y2, x = 4
9. xy = 6, y = 2, y = 6, x = 6
In exercise 10-12, find the work done for the following question
10. Determine the work done in lifting a 100 pound bag of teff 10 feet.
11. A force of 25 pounds is required to slide a cement block on a plank in a construction project.
The plank is 12 feet long; determine the work done in sliding the block along the length of the
plank.
12. A force of 60 pounds stretches a spring 1 foot. How much work is done in stretching the
spring from 9 inches to 15 inches?
In exercise 13-15, find the arc length of the graph of the given function over the indicated interval
3
13. f(x) = x 2
- 1, on the interval [0, 4]
2
15. f(x) = cos x, on the interval [0, ]
2
156
Unit 8: exponential and logarithmic function
CONTENTS
8.0 Aims and Objectives
8.1 Introduction
8.2 Definition, Derivation and Integral of Exponential, Logarithmic and Hyperbolic Function
8.1 INTRODUCTION
In this unit you will develop more general logarithmic and exponential function by using the theory
of calculus. These functions are called natural logarithmic and exponential function and both are
continuous and differentiable throughout their domain.
Basically there are two approaches in developing the natural logarithmic and exponential
functions. Regardless of the approaches taken, the natural logarithmic and exponential functions
will turn out to be inverses of each other. We can define either function first and obtain the other
function as the inverse of the first.
157
8.2 definition, derivatives and integrals of exponential, logarithmic
DEFINITION 8.2.1
The natural logarithm of a positive number x, written as ln x is the value of an integral and is
defined as
x
1
ln x = t dt ,
1
>0
x
1
If x > 1, ln = t dt
1
give the area of the region R, bounded by the graph of f (t) = 1/t and the x-axis
between t = 1 and t = x
158
If x < 1 , lnx is the negative of the area bounded by the graph of f (t) = 1/t, the x-axis between t =
x and t = 1
Example 8.2.1 Apply the rules discussed above to rewrite the following
a) ln 4 + ln sinx = ln (4 sin x)
x 1
b) ln = ln (x + 1) – ln (4 sin x)
2 x 3
1
c) ln sec x = ln = ln 1 – ln cos x
cos x
1
d) ln 3
x 1 = ln (x + 1)1/3 = ln (x + 1)
3
Note the graph of lnx has the following properties
- the domain of y = lnx is (0, +)
- the function y = lnx is increasing for all x in its domain.
- The graph of y = lnx is concave down on (0, )
1
1
- Ln1 = t dt
1
=0
lim ln x , lim ln x
-
x x 0
- the range of ln x is (- )
159
The graph of the natural logarithm function
Derivative of y = lnx
d 1
By the first part of fundamental theorem of calculus we have lnx =
dx x
If U is a differentiable function of x whose values are positive, so that lnu is defined,
d 1 du
then lnu =
dx u dx
Example 8.2.2
d
Evaluate ln2x
dx
Solution: -
d 1 d
(ln2x) = (2x)
dx 2 x dx
1
= .2
2x
1
=
x
d 1 d 1 1
Note lnax = (ax) = - a = , for any number a > 0
dx dx dx dx x
Example 8.2.3
d
Evaluate ln (x2 + 3)
dx
160
Solution
d 1 d
ln (x2 + 3) = 2 . (x2 + 3)
dx x 3 dx
1
= 2 . 2x
x 3
2x
= 2
x 3
1
The integral du
u
and
u ( x) 1
u ( x) dx u du ln u ( x) c , u ( x) 0
Example 8.2.4
2x
Evaluate x 2
5
dx
Solution: -
Let u = x2 – 5
du = 2x dx
2x du
x 2
5
dx = u
ln u c ln x 2 5 c
Example 8.2.5
1 cos x
Evaluate x sin x dx, x + si n x 0
161
= ln | x + s in x | + c
Example 8.2.6
x 1
Evaluate x 2
2x
dx
Solution let u = x2 + 2x
du = (2x + 2) dx = 2 (x + 1) dx
du
= (x + 1) dx
2
x 1 du 1 du 1
x 2
2x
dx = u
2 u
ln u c
2
1
= ln x 2 2 x c
2
Note the function f (x) = lnx is a continuous, increasing function whose range is the entire real
no., there is exactly one number x for which lnx = 1, we denote this number by the letter e, ie
e
1
1= t dt
1
- The number e is an irrational number, and the decimal representation for e correct to 12
decimal places is known to be e 2.718281828459
162
dx ln 2 x 1
d) x ln x e) x dx f)
x 1 x dx
e2
1
g) x ln x
e
Definition 8.2.2
If x is any real number, we define y = ex of and only if x = lny.
Since lnx and ex are inverses of one another, we have
elnx =x x > 0
lnex = x x
Laws of exponents for ex
For all numbers x, x1 and x2
1. ex1 . ex2 = ex1 + x2
1
2. e-x =
ex
e x1
3. x2
e x1 x 2
e
4. e x1 x2
e x1 x2 e x2
x1
Example 8.2.7 use the lows of exponents to rewrite the given expression.
a) e x ln 2 b) e-lnx
1 1
Solution ex + ln 2 = ex . eln2 = 2ex Solution e-lnx = ln x
e x
c)
e2x
e
d) e 3
x
Solution
e2x
e
e 2 x 1 Solution e 3
x
= e3x
163
d x
dx
e ex
Example 8.2.8
Evaluate
d
dx
5e x
Solution
d
dx
5e x =5
d x
dx
e
= 5ex
Example 8.2.9
Find the derivative of the following function
a) y = e-x b) y = esinx c) y = e6x d) y = ex sinx
Solution
a)
dx
d x
e ex
d
dx
1 b)
dx
e
d sin x
e sin x
d
dx
(sin x)
c)
dx
d 6x
e e6x
d
dx
(6 x ) d)
dx
e
d x sin x
e x sin x
d
dx
x sin x
Integrals involving ex
e dx e x c
x
e u ( x) dx e u du e u c
u ( x)
164
Example 8.3.10
Find the following integral
a) e 3 x dx b) x 2 e x 1
3
dx
Solution Solution
Let u = -3x let u = x3 + 1
du = -3dx du = 3x2 dx
du du
dx = = x2 dx
3 3
eu eu
e dx = 3 du x e 3 du
3 x 2 x 3
1
dx =
1 u 1 u
3 3
= e du = e du
1 u 1
= e c = eu c
3 3
1 3 x 1 3
= e c = e x 1 c
3 3
ln 3
x2
d) 2 x e dx e) e
x
dx
ln 2
165
e
1
Note the number e is difficult to approximate from the defining property lne = x dx 1 , since it
1
appears as a limit of integration. But this same number can be obtained in different way as the
limit.
x
lim r
e= 1
x x
It is easier to approximate e as this limit, since we need only calculate values of the function
x
1
1 for large x.
x
x
lim 1
r
e = 1 , rR
x x
n
1
Table 8.3.1 Approximating e 2.7182818, --- using 1
n
n 1
n n 1
n
1 1
n n
1 2.000000 500 2.715569
5 2.488320 1000 2.716925
20 2.653298 2500 2.717742
50 2.691588 5000 2.718016
100 2.704814 10,000 2.718160
250 2.712865 100,000 2.718442
Example 8.3.11
Evaluate the following limits
lim 1 x
a) 1 =
x x
166
Solution
1
x
lim 1
1 = e-1 =
x x e
lim 3 x
b) 1
x x
Solution
lim 3
1 = e3
x x
2x
lim 6
c) 1
x x
Solution
2
6 x
lim
x
1 e
6
2
e12
x
x
lim 1
d) 1
x x2
Solution
x
lim 1
x
lim 1 1
1 2 = 1 1
x x x x x
x x
lim 1 lim 1
= 1 1
x x x x
= e-1 . e
1
= -e
e
=1
167
The hyperbolic function
Definition 8.2.3
The hyperbolic sine and cosine functions, sin hx and cos hx are defined as:
1 x
sinhx = (e – e-x)
2
1 x
coshx = (e + e-x)
2
Note – coshx represents the average of exponential growth and exponential decay.
- sinhx represents half the difference between exponential growth and exponential decay.
Definition 8.2.4
The hyperbolic tangent, cotangent, secant and cosecant function are defined as:
sinh x e x e x
tanhx =
cosh x e x e x
cosh x e x e x
coshx =
sinh x e x e x
1 2
sechx = x
cosh x e e x
1 2
cschx = x x
sinh x e e
168
The hyperbolic functions so far discussed are a simple combination of the differentiable function
ex and e-x and hence have derivatives in their domain.
d
sinh u cosh u du
dx dx
d
cosh u sinh u du
dx dx
d
tanh u sec h 2 u du
dx dx
d
coth u csc h 2 u du
dx dx
d
sec h u sec hu tanh u du
dx dx
d
csc h u csc hu coth u du
dx dx
sinh du = cosh u + c
cosh du = sinh u + c
sec h u du = tanh u + c
2
csc h u du = -coth u + c
2
169
Example 8.2.12
Evaluate
d
dt
tanh 1 t 2
Solution
d
dt
tanh 1 t 2 = sec h 2 1 t 2
d
dt
tanh 1 t 2
1
= sec h 2 1 t 2
1 t 2
Example 8.2.13
Evaluate coth 5 x dx
Solution
cosh sx
coth 5x dx = sinh sx dx
Let u = sin 5x
du = s cosh 5x dx
du
= cosh 5x dx
5
cosh 5 x du 1 du 1
sinh 5x dx = 5u
5 u
ln u c
5
1
= ln sin 5 x c
5
Example 8.2.14
1
Evaluate sinh 2 x dx
0
Solution
cosh 2 x 1
1 1
sinh x dx =
2
dx
0 0
2
170
1
1
=
2 (cosh 2 x 1) dx
0
1 sinh 2 x 1
= x
2 2 0
sinh 2 1
= -
4 2
= 0.40672
Example 8.2.15
ln 2
4e
x
Evaluate sinh x dx
0
Solution
ln 2 ln 2
e x ex
4 e sinh x dx = 4 e x dx
x
0 0 2
e2x e0
ln 2
=4
0
2
dx
= e2x – 2x ln20
= (e2ln2 – 2ln2) – (e0 – 1)
= e2ln2 – 2ln2 – 1
= 4 – 2 ln2 – 1
1.6137
171
Check your progress 8.2.4
1.Find the derivative of the given function
6 1
dx 1 sinh x
d)
2 x2 4
e)
0 9 x 2 25
dx f) cosh x dx
Let y be a quantity that we are interested in, that increases and decreases at the rate at any given
time t is proportional to the amount present. Let y0 be the initial amount at time t = 0 then we can
find y as a function of t by using the initial condition given.
i.e.
dy
ky , y = y0 when t = 0
dt
k is growth constant
k > 0, if y > 0 and increasing
k < 0 if y < 0 and decreasing
dy
ky
dt
1 dy 1
k dy = kdt
y dt y
1
y dy kdt
ln |y|= kt + c
|y| = ekt + c
|y| = ekt . ec
172
y = (ec ekt)
y = A ekt , A = ec
Consider the initial amount at t = 0
y0 = Ae(0)
y0 = A
y = y0 ekt
Example 8.3.1
dy
One model for the way disease spread assumes that the rate at which the number of infected
dt
people changes is proportional to the number y. the more infected people there are the faster the
disease will spread. The fewer there are, the slower it will spread.
Suppose that is the course of any given year the number of cases of a disease is reduced by 20% if
there are 10,000 cases today, how many years will it take to reduce the number to 1000?
Solution
We use the formula y = y0 ekt
173
8000
ek =
10,000
ek = 0.8
lnek = ln (0.8)
k = ln (0.8)
at any given time t y = 10,000 e (ln0.8)t
Step 3 The value of t that makes y = 1000. we set y equal to 1000 in the equation produced in step
2 above
1000 = 10,000 ln(0.8)t
0.1= e(ln 0.8)t
(ln (0.8) t = ln 0.1
ln 0.1
t= 10.32 years
ln 0.8
It will take little more than 10 years to reduce the number of cases to 1000.
Example 8.3.2
Assume that the rate of growth of a population of fruit flies is proportional to the size of the
population at each instant of time. If 100 fruit flies are present initially and 300 are presents after
10 days. How many will be present after 15 days?
Solution
dy
Since it is stated that population size Y satisfies the equation ky
dt
ie. y (t) = y0ekt
We are given that y = 100 when t = 0, ie.
y0 = 100
It is also given that y = 300 when t = 10
Hence 300 = 100 ek(10)
e10k = 3
10k = ln3
ln 3
k=
10
174
y (t) = 100 e(ln3/10)t
y (15) = 100 e(ln3/10)15
= 100 e3/2 ln3
520 fruit flies
8.5 SUMMARY
ax
- a x dx c
ln a
* The law of Exponential change
175
Y = Y0ekt
Growth: k > 0, Decoy: k < 0
When x is the rate constant of the equation.
e x 1
d) f (x) = ln (sinx – x cosx) e) f (x) = xe-x f) f (x) =
ex
g) esinx h) f (x) = xe-3lnx i) f (x) = x coshx
e 3
cos x 1 x
d) sin x dx e) 1 x dx f) x
2
2
1
dx
x
e
g) e 2 x dx h) e 2 x 6 dx i) x
dx
1
j) cos x e k) xe 1 x dx
2
sin x
dx
0
2
l) x sinh x 2 dx m) tanh 2 x sec h 2 x dx n) e x sin x dx
1
176
8.6 ANSWERS TO CHECK YOUR PROGRESS QUESTIONS
1 1
2. a) – ln /1 – x/ + c b) ln x 2 2 x c c) ln x 3 9 x c
2 3
d) ln /lnx/ + c e)
1
3
ln x c
3
f) –2 ln 1 x c
g) ln 2
x
e
1. a) 3 e3x b) 2 x
c) (2x – 1) ex2 – x d) ex (sinx + cosx)
1 3x
2. a) e – 5e-x + c b) –e1/x + c c) 8 e(x + 1)
3
d) –e-x2 + c e) 1
1. a) e b) 1/e
177
Answers to check your progress 8.2.4
sinh x
c) 2 (cosh (4x)-1/2 sinh (4x) d)
cosh 2 x
2 5
2. a) ln 1
b) x tanh (2 x) c c)
e 2
e 2
2
1 2 2 8
1 3 34
d) ln 3 8 e) ln f) ln /coshx/ + c
3 5 5
178
Unit 9: Trigonometric functions and it’s Inverse
CONTENTS
9.0 Aims and Objectives
9.1 Introduction
9.2 Derivatives of
9.2.1 Trigonometric Function
9.2.2 Inverse of Trigonometric Function
9.3 Integration of
9.3.1 Trigonometric Function
9.3.2 Inverse of Trigonometric Function
9.4 Summary
9.5 Answer to Check Your Progress
9.6 Model Examination Questions
In this section we will compute the derivatives of sin x, cos x and related functions. Thus at the
end of this unit, you should be able to:
evaluate the derivatives of trigonometric functions and it’s inverse
find the numerical values for different trigonometric ratios
limsin x
prove = 1 and other trigonometric function
x0 x
integrate trigonometric functions and its inverse
use identity properties of trigonometric function to find the derivative and integration of
trigonometric functions.
9.1 INTRODUCTION
Trigonometric functions arise in geometry, and applicable to the study of sound, motion of
pendulum and many other problems containing rotations.
179
For this reason we should know the basic identity property of trigonometric rations. To list some
of them
Sin2x + cos2x = 1
1 + tan2x = sec2x
1 + cot2x = csc2x
Sin (x y) = sin x cos y + cos x sin y
Cos (x y) = cos x coy + sin x sin y
Sin (-x) = -sin x
Cos (-x) = cos x
Hence for the next section, we use these and other identities to find the derivative and integration
of trigonometric functions.
9.2 DERIVATIVES OF
Theorem 9.2.1.1
limsin x
a) =1
x0 x
lim cos x 1
b) =0
x0 x
Proof: Consider the following figure
180
For 0 x the following holds true
2
a) the area of OAD
D (cos x sin x)
A(1, 0)
1
Area of OAD = bh
2
1
= (1 unit) (sin x)
2
1
= (1 unit) (sin x)
2
sin x
=
2
sin x
Area of OAD =
2 ---------(1)
x
Area of sector OAD = ------------(2)
2
181
1
c) Area of OAC = bh
2
1
= (1 unit) (tan x)
2
tan x
=
2
tan x
Area of OAD = ----------(3)
2
1, for 0 |x|
sin x
Cos x
x 2
182
limsin x
1 1
x0 x
limsin x
=1
x0 x
lim cos x 1
b) =0
x0 x
lim cos x 1 lim (cos x 1)(cos x 1)
Proof: =
x0 x x0 x(cos x 1)
lim cos 2 x 1
=
x 0 x(cos x 1)
lim sin 2 x
= - since sin2x + cos2x = 1
x 0 x(cos x 1)
lim (sin x) (sin x)
= -
x0 x(cos x 1)
limsin x lim sin x lim 1
=- . .
x0 x x0 x 0 cos x 1
= -(1) (0) 12
=0
lim cos x 1
=0
x0 x
d
Theorem 9.2.1.2. The derivative of sin x is cos x sin x = cos x and the derivative of cos x is –
dx
d
si n x cos x = - sin x
dx
d lim f ( x h) f ( x )
Proof: - We have to use the definition of derivative f (x) =
dx h0 h
d lim sin( x h) sin x
Now si n x =
dx h0 h
lim sin x cos h cos x sin h sin x
=
h0 h
lim sin x cos h sin x cos x sin h
=
h0 h
183
lim sin x(cos h 1) cos x sin h
=
h0 h
lim sin x lim cos h 1 lim cos x lim sin h
= . + .
h0 h0 h h0 h0 h
= (sin x) (0) + (cos x) (1)
= 0 + cos x
= cos x
d
Therefore sin x = cos x
dx
d
Proof: cos x = -sin x
dx
d lim cos( x h) cos x
cos x =
dx h0 h
lim cos x cos h sin x sin h cos x
=
h0 h
lim cos x cos h cos x sin x sin h
=
h0 h
lim cos x(cos h 1) sin x sin h
=
h0 h
lim cos x, lim cos h 1 lim sin x, lim sin h
= - .
h0 h0 h h0 h0 h
= (cos x) (0) – (sin x) (1)
= 0 – si n x
= -sin x
d
Therefore cos x = -sin x
dx
Theorem 9.2.1.3
d
a) tan x = sec2x
dx
d
b) cot x = -csc2x
dx
d
c) sec x = sec x tan x
dx
184
d
d) csc x = -csc x cot x
dx
Proof: -
d d sin x
a) tan x =
dx dx cos x
d cos x(cos x) sin x( sin x)
tan x = , Quotient rule
dx (cos x) 2
cos 2 x ( sin 2 x)
=
cos 2 x
cos 2 x sin 2 x
= , Since sin2x + cos2x = 1
cos 2 x
1
=
cos 2 x
= sec2x
d
Hence, tan x = sec2x
d
d d cos x
b) cot x =
dx dx sin x
sin x( sin x) cos x(cos x)
=
(sin x) 2
sin 2 x cos 2 x
=
(sin x) 2
(sin 2 x cos 2 x)
=
sin 2 x
1
=
sin 2 x
= - csc2x
d
Hence cot x = -csc2x
dx
C and d are exercises
185
Example 9.2.1.1. Compute the derivative of the following functions
a) sin 3x c) cos (x – 3)
sin x
b) d) sin (x3 – 2x + 6)
x
Solution:
a) Let U = 3x and f(u) = sin u. Using the chain rule we have
d d
sin 3x = f(u)
dx dx
df du du
= . , since =3
du dx dx
= cos u . (3)
= 3 cos u
= 3 cos (3x)
= 3 cos 3x
d
sin 3x = 3 cos 3x
dx
d sin x (sin x)( x) (sin x)( x)
b) = Quotient rule
dx x x2
(cos x)( x) (sin x)(1)
=
x2
x cos x sin x
=
x2
d sin x x cos x sin x
=
dx x x2
d
c) cos (x – 3), let U = x – 3 and f(u) = cos u
dx
using chain rule, we have
d d
cos (x – 3) = f(u)
dx dx
df du du
= . since =1
du dx dx
186
= (-sin u) (1)
= -sin u
= -sin (x – 3)
d
cos (x – 3) = -sin (x – 3)
dx
d
d) sin (x3 – 2x + 6)
dx
Let U = x3 – 2x + 6 and f(u) = sin U
d df
sin (x3 – 2x + 6) = (u)
dx dx
df du du
= . , since = 3x2 – 2
du dx dx
= (cos U) (3x2 – 2)
= (3x2 – 2) cos (x3 – 2x + 6)
d
sin (x3 – 2x + 6) = (3x2 – 2) cos (x3 – 2x + 6)
dx
187
a) arc sin (-½)
Solution: by definition y = arc sin (-½) iff
Sin y = -½
y=
6
b) arc cos 0
Solution: by definition y = arc cos 0 iff
Cos y = 0
y=
2
Remark:
Inverse properties
a) sin (arc sin x) = x
b) tan (arc tan x) = x
c) sec (arc sec x) = x, and similar for the rest.
d U
(arc cos u) =
dx 1U 2
d U
(arc cot u) =
dx 1U 2
d U
(arc csc u) =
dx U U 2 1
Let us prove the first theorem and the rest is left as an exercise
d U
a) (arc sin u) =
dx 1U 2
Proof: Let f(x) = sin x and f-1(x) = g(x) = arc sin x
188
d 1
[g(x)] = 1
dx f [ g ( x)]
1
=
cos(arc sin x)
Now let m = arc sin x
d 1
Thus [g(x)] =
dx cos
1
= , Since cos2m + sin2m = 1
1 sin m
2
cos2m = 1 – sin2m
d 1
and [ arc sin x] =
dx 1 (sin 2 m)
1
=
1 [sin(arc sin x)]2
1
= , since sin (arc sin x) = x
1 x2
d 1
:- arc sin x =
dx 1 x2
d U
If U is differentiable function of x, using chain rule we have (arc sin u) = where U/ =
dx 1U 2
du
dx
Example 9.2.2.2. Find the derivatives of
a) f(x) = arc sin 2x
Solution:
Let U = 2x and f(u) = arc sin u.
d df
(arc sin 2x) = (u)
dx dx
df du du
= . , since = (2x)/ = 2
du dx dx
189
1
= (2)
1 u2
2
= , but u = 2x
1 u2
d 2 2
Thus (arc sin 2x) = =
dx 1 (2 x) 2 1 4x 2
2
=
1
2
1 u
2
1
= but u = x
1 u 2 2
1
=
1 x 2
2
1
=
1 x
2
4
1
=
4 x2
4
1
=
4 x2
2
2
=
4 x2
190
d 2
Thus (2 arc cos x ) =
dx 2 4 x2
9.3 INTEGRATION
191
a) 2 cos 2x dx
Solution:
Let u = 2x
du = 2dx
du
= dx
2
du
dx =
2
du
Now 2cos 2x dx = 2 cos u
2
= cos u du
= s in u + c
= sin 2x + c
Thus 2 cos 2x dx = sin 2x + c
b) tan x dx
Solution:
sin x
tan x dx = cos x dx
Let u = cos x
du = - sin x dx
du
= dx
sin x
du
dx =
sin x
sin x
Now tan x dx = cos x dx
sin x du
= .
U sin x
du
= U
du
=
U
= -ln |u| + c
192
= -ln |cos x| + c
Therefore tan x dx = -ln |cos x| + c
du u
2. a 2
u 2
= arc tan + c
a
du 1 u
3. u u2 a2
=
a
arc sec
a
+c
193
dx dx
2 9x 2
= 2 3x
2 2
1 1 3x
=
3 2
arc tan c
2
1 3x
= arc tan c
3 2 2
4. sin 2x cos 2x dx
1
6. 1 4x 2
dx
t
7. 1 t4
dt
194
The derivatives of trigonometric functions and its inverse.
d
tan x = sec2x
dx
d
sec x = sec x tan x
dx
d
cot x = -csc2x
dx
d
cos cx = -csc x cot x
dx
d
sin x = cos x
dx
d
cos x = -sin x
dx
d 1
arc sin x =
dx 1 x2
d 1
arc cos x =
dx 1 x2
d 1
arc tan x =
dx 1 x2
d 1
arc sec x =
dx x x2 1
d 1
arc cot x =
dx 1 x2
d 1
arc csc x =
dx x x2 1
195
sin x dx = -cos x + c
cos x dx = sin x + c
tan x dx = -ln |cos x| + c
cot x dx = ln |sin x| + c
sec x dx = ln |sec x + tan x| + c
cscx dx = -ln |csc x + cot x| + c
dx
a2 x2
= arc sin x
a +c
dx 1 x
x x2 a2
=
a
arc sec
a
+c
dx 1 x
a 2
= arc tan + c
x a a
2. 3 sin x + c
2
3. 5 sin x + c
5
4. 1 sin2 2x + c
4
1
5. arc sin 3x + c
3
196
1
6. arc tan 2x + c
2
1
7. arc tan t2 + c
2
ex
13. dx
1 e2x
x3
14. x 2 1 dx
1
15. 9 ( x 3) 2
dx
1
16. x 2 4x
dx
197
198